Иррациональные уравнения и способы их решения: Методы решения иррациональных уравнений

Содержание

Методы решения иррациональных уравнений


Я бы почувствовал настоящее

удовлетворение лишь в том случае,

если бы смог передать ученику гибкость ума,

которая дала бы ему в дальнейшем

возможность самостоятельно решать задачи.

У.У.Сойер.



Определение. Уравнение с одной
переменной называют иррациональным, если хотя бы
одна из функций или содержит переменную под знаком
радикала.

При решении иррациональных уравнений
необходимо установить область допустимых
значений переменных, исходя из условия, что все
радикалы, входящие в уравнение, должны быть
арифметическими.


1. Метод пристального взгляда

Этот метод основан на следующем теоретическом
положении: “Если функция возрастает в области
определения и число входит в множество значений, то уравнение имеет
единственное решение
.

Для реализации метода, основанного на этом
утверждении требуется:

а) Выделить функцию, которая фигурирует в
уравнении.

b) Записать область определения данной функции.

c) Доказать ее монотонность в области
определения.

d) Угадать корень уравнения.

t) Обосновать, что других корней нет.

f) Записать ответ.



Пример 1. .

Наличие радикалов четной степени говорит о том,
что подкоренные выражения должны быть
неотрицательными. Поэтому сначала найдем
область допустимых значение переменной .

Очевидно, что левая часть уравнения не
существует ни при одном значении неизвестного . Таким
образом, вопрос о решении уравнения снимается –
ведь нельзя же осуществить операцию сложения в
левой части уравнения, так как не существует сама
сумма. Каков же вывод? Уравнение не может иметь
решений, так как левая часть не существует ни при
одном значении неизвестного .



Пример 2.

Рассмотрим функцию .

Найдем область определения данной функции:

Данная функция является монотонно
возрастающей.

Для
эта функция будет принимать наименьшее значение
при , а
далее только возрастать.. Число 5 принадлежит
области значения, следовательно, согласно
утверждению .

Проверкой убеждаемся, что это действительный
корень уравнения..


2. Метод возведения обеих частей уравнений в
одну и ту же степень.



Теорема.

Если возвести обе части уравнения (1) в
натуральную степень , то уравнение (2)
является следствием уравнения (1).

Доказательство. Если выполняется числовое
равенство , то по свойствам степени выполняется
равенство , т.е. каждый корень уравнения (1) является
и корнем уравнения (2), это значит, что уравнение (2)
является следствием уравнения (1).

Если ,
то справедливо и обратная теорема. В этом случае
уравнения (1) и (2) равносильны.

Если ,
равенство справедливо, если выполняется хотя бы
одно из равенств и . Значит уравнения (1) и (2) в этом
случае не равносильны. Поэтому, если в ходе
решения иррационального уравнения
приходилось возводить обе его части в степень с
четным показателем, то могли появиться
посторонние корни. Чтобы отделить их, проверки
можно избежать, введя дополнительное требование . В этом
случае уравнение равносильно системе . В
системе отсутствует требование ,
обеспечивающее существование корня степени , т.к. оно
было бы излишним в связи с равенством .



Пример 1.



,

,

.

Ответ:

Если в уравнение входят несколько радикалов, то
их можно последовательно исключать с помощью
возведения в квадрат, получая в итоге уравнение
вида
При этом полезно учитывать область допустимых
значений исходного уравнения.



Пример 2. 

Ответ:


3. Решение уравнений с использованием замены
переменной.



Введение вспомогательной переменной в ряде
случаев приводит к упрощению уравнения. Чаще
всего в качестве новой переменной используют
входящий в уравнение радикал. При этом уравнение
становится рациональным относительно новой
переменной.



Пример1. 

Пусть тогда исходное уравнение примет вид:

,
корни которого и Решая уравнение , получаем и

Ответ:

В следующих примерах используется более
сложная замена переменной.



Пример 2



Перенесем в левую часть все члены уравнения и
произведем дополнительные преобразования: .

Замена приводит уравнение к виду корнями
которого являются и

Осталось решить совокупность двух уравнений:

Ответ:


4. Метод разложения на множители выражений,
входящих в уравнение.

Теорема.



Уравнение , определенное на всей числовой оси,
равносильно совокупности уравнений

Пример1.

При
уравнение принимает вид: которое равносильно
совокупности двух уравнений:

Ответ:

Выделить общий множитель часто бывает очень
трудно. Иногда это удается сделать после
дополнительных преобразований. В приведенном
ниже примере для этого рассматриваются попарные
разности подкоренных выражений.



Пример 2.

Если внимательно посмотреть на уравнение, то
можно увидеть, что разности подкоренных
выражений первого и третьего , а также второго и
четвертого членов этого уравнения равны одной и
той же величине

В таком случае далее следует воспользоваться
тождеством:

Уравнение примет вид:

или

Корень уравнения т. е. число при подстановке в исходное
уравнение дает верное равенство.

Уравнение не имеет решений, так как его левая часть
положительна в своей области определения.

Ответ:


5. Метод выделения полных квадратов при решении
иррациональных уравнений.


При решении некоторых иррациональных
уравнений полезна формула



Пример 1.



Преобразуем уравнение следующим образом:

или



Обозначим и решим полученное уравнение

методом интервалов.

Разбирая отдельно случаи ,
находим,

что решениями последнего уравнения являются .

Возвращаясь к переменной , получаем неравенства

Ответ:


6. Метод оценки.



Этот способ применим в том случае, когда
подкоренные выражения представляют собой
квадратный трехчлен, не раскладывающийся на
линейные множители. Поэтому целесообразно
оценить левую и правую части уравнения.



Пример 1.

Оценим обе части уравнения:

,

,

Левая часть уравнения существует при всех
значениях переменной , не меньших 5, а правая – при всех
значениях, не больших 5, следовательно, уравнение
будет иметь решение, если обе части уравнения
одновременно равны 5, т. е. справедлива следующая
система:

Корнем второго уравнения системы является
число

Проверим, является ли это число корнем второго
уравнения:

.

Ответ:



Пример 2.



Для всех имеем

Используя неравенство Коши, можем записать:

причем равенство достигается при и

Таким образом, -корень исходного уравнения.

Ответ:


7. Иррациональные уравнения, содержащие
степени выше второй.

Если уравнение имеет вид то его можно решить , возводя
обе части этого уравнения в степень . Полученное
уравнение при нечетном равносильно данному уравнению, а при
четном является
нго следствием, аналогично рассмотренному выше
случаю при



Пример 1



Возведем обе части уравнения в куб:

или

которое
равносильно совокупности двух уравнений:

Ответ:

При решении иррациональных уравнений очень
часто пользуются следующим приемом.

Если
то

В последнем равенстве заменяют на и
получают

Далее легко избавиться от кубической
иррациональности , возводя обе части в куб.



Пример 2.


Здесь, очевидно,

Возведем в куб обе части уравнения, получим:

,

или

или

или

или

Проверка подтверждает, что это корень
уравнения.

Ответ:

Замечание.

Замена в конкретном примере левой части на
правую, вообще говоря , неправомерна –ведь нам
неизвестно ни одно значение , при котором это
уравнение превращается в верное числовое
равенство. Возможно, таких решений нет вообще.
Допуская в практических действиях такую замену,
мы фактически расширяем возможное множество
решений. Поэтому все найденные решения следует
проверять и только те, которые превращают
исходное уравнение в верное равенство, следует
записать в ответ.

От того, что школьник решит лишний десяток
задач, умнее и сообразительнее он не станет,
Результат обучения оценивается не количеством
сообщаемой информации, а качеством ее усвоения.
Это качество будет выше, если на один и тот же
пример посмотреть с разных сторон. Решение задач
разными способами способствует развитию
активного мышления учащихся. Хорошую почву для
этого дает решение примеров разными способами.



Пример 3. Способ 1.

(1)

Возведем обе части уравнения в куб:

Группируя, получаем:

Используя равенство (1) имеем:

или

или

или


корни которого

Ответ:

Способ 2.

Иногда полезно ввести не одну вспомогательную
переменную, а несколько, сводя исходное
уравнение к системе уравнений.

Пусть Тогда

Таким образом справедлива следующая система:

Возвращаясь к переменной находим

Ответ:

В следующем примере введение вспомогательной
переменной сводит исходное уравнение к
однородному.



Пример 4.

Положим

Тогда исходное уравнение примет вид:

Поскольку при котором переменная
обращается в нуль, не является решением
исходного уравнения ( в чем можно убедиться
подстановкой), делим обе части уравнения на


решая которое , находим:

Осталось решить уравнения и

Корнями этих уравнений являются числа

Ответ:



Пример 5.

Область допустимых значений задается
неравенством

Преобразуем уравнение следующим образом:

Один корень этого уравнения

Для решения второго уравнения положим

и решим

Корни этого уравнения

Последний корень не принадлежит указанному
промежутку, поэтому, решая уравнение , получим

Ответ :

Методы решения иррациональных уравнений

Методы решения иррациональных уравнений.

Предварительная подготовка к уроку: учащиеся должны уметь решать иррациональные уравнения различными способами.

За три недели до данного занятия учащиеся получают домашнее задание №1: решить различные иррациональные уравнения. (Учащиеся самостоятельно находят по 6 различных иррациональных уравнений и решают их в парах.)

За одну неделю до данного занятия учащиеся получают домашнее задание №2, которое выполняют индивидуально.

1. Решить уравнение различными способами.

2. Оценить достоинства и недостатки каждого способа.

3. Оформить запись выводов в виде таблицы.

№ п/п

Способ

Достоинства

Недостатки

       

Цели урока:

Образовательная: обобщение знаний учащихся по данной теме, демонстрация различных методов решения иррациональных уравнений, умения учащихся подходить к решению уравнений с исследовательских позиций.

Воспитательная: воспитание самостоятельности, умения выслушивать других и общаться в группах, повышение интереса к предмету.

Развивающая: развитие логического мышления, алгоритмической культуры, навыков самообразования, самоорганизации, работы в парах при выполнении домашнего задания, умений анализировать, сравнивать, обобщать, делать выводы.

Оборудование: компьютер, проектор, экран, таблица «Правила решения иррациональных уравнений», плакат с цитатой М.В. Ломоносова «Математику уже затем учить следует, что она ум в порядок приводит», карточки.

Правила решения иррациональных уравнений.

Тип урока: урок-семинар (работа в группах по 5-6 человек, в каждой группе обязательно есть сильные ученики).

Ход урока

I. Организационный момент

(Сообщение темы и целей урока)

II. Презентация исследовательской работы «Методы решения иррациональных уравнений»

(Работу представляет учащийся, который ее проводил.)

III. Анализ методов решения домашнего задания

(По одному учащемуся от каждой группы записывают на доске предложенные ими способы решения. Каждая группа анализирует один из способов решения, оценивает достоинства и недостатки, делает выводы. Учащиеся групп дополняют, если это необходимо. Оценивается анализ и выводы группы. Ответы должны быть четкими и полными.)

Первый способ: возведение обеих частей уравнения в одну и ту же степень с последующей проверкой.

Решение. Возведем обе части уравнения в квадрат:

Снова возведем обе части уравнения в квадрат:

Отсюда

Проверка:

1. Если х=42, то , значит, число 42 не является корнем уравнения.

2. Если х=2, то , значит, число 2 является корнем уравнения.

Ответ: 2.

№ п/п

Способ

Достоинства

Недостатки

1

Возведение обеих частей уравнения в одну и ту же степень

1. Понятно.

2. Доступно.

1. Словесная запись.

2. Сложная проверка.

Вывод. При решении иррациональных уравнений методом возведения обеих частей уравнения в одну и ту же степень необходимо вести словесную запись, что делает решение понятным и доступным. Однако обязательная проверка иногда бывает сложной и занимает много времени. Этот метод можно использовать для решения несложных иррациональных уравнений, содержащих 1–2 радикала.

Второй способ: равносильные преобразования.

Решение: Возведем обе части уравнения в квадрат:

Ответ:2.

№ п/п

Способ

Достоинства

Недостатки

2

Равносильных преобразований

1. Отсутствие словесного описания.

2. Нет проверки.

3. Четкая логическая запись.

4. Последовательность равносильных переходов.

1. Громоздкая запись.

2. Можно ошибиться при комбинации знаков системы и совокупности.

Вывод. При решении иррациональных уравнений методом равносильных переходов нужно четко знать, когда ставить знак системы, а когда – совокупности. Громоздкость записи, различные комбинации знаков системы и совокупности нередко приводят к ошибкам. Однако последовательность равносильных переходов, четкая логическая запись без словесного описания, не требующая проверки, являются бесспорными достоинствами данного способа.

Третий способ: функционально-графический.

Решение.

Рассмотрим функции и .

1. Функция степенная; является возрастающей, т.к. показатель степени – положительное (не целое) число.

Найдем область определения функции D(f).

Составим таблицу значений x и f(x).

x

1,5

2

3,5

6

f(x)

0

1

2

3

2. Функция степенная; является убывающей.

Найдем область определения функции D(g).

Составим таблицу значений x и g(x).

x

0

2

6

g(x)

4

3

1

-1

Построим данные графики функций в одной системе координат.

Графики функций пересекаются в точке с абсциссой Т.к. функция f(x) возрастает, а функция g(x) убывает, то решение уравнения будет только одно.

Ответ: 2.

№п/п

Способ

Достоинства

Недостатки

3

Функционально-графический

1. Наглядность.

2. Не нужно делать сложных алгебраических преобразований и следить за ОДЗ.

3. Позволяет найти количество решений.

1. словесная запись.

2. Не всегда можно найти точный ответ, а если ответ точный, то нужна проверка.

Вывод. Функционально-графический метод является наглядным, позволяет найти количество решений, но применять его лучше тогда, когда легко можно построить графики рассматриваемых функций и получить точный ответ. Если ответ приближенный, то лучше воспользоваться другим методом.

Четвертый способ: введение новой переменной.

Решение. Введем новые переменные, обозначив Получим первое уравнение системы

Составим второе уравнение системы.

Для переменной :

,

Для переменной

Поэтому

Получим систему двух рациональных уравнений, относительно и

Вернувшись к переменной , получим

Ответ: 2.

№п/п

Способ

Достоинства

Недостатки

4

Введение новой переменной

Упрощение – получение системы уравнений, не содержащих радикалы

1. Необходимость отслеживать ОДЗ новых переменных

2. Необходимость возврата к исходной переменной

Вывод. Этот метод лучше применять для иррациональных уравнений, содержащих радикалы различных степеней, или одинаковые многочлены под знаком корня и за знаком корня, или взаимообратные выражения под знаком корня.

– Итак, ребята, для каждого иррационального уравнения необходимо выбирать наиболее удобный способ решения: понятный. Доступный, логически и грамотно оформленный. Поднимите руку, кто из вас при решении этого уравнения отдал бы предпочтение:

1) методу возведения обеих частей уравнения в одну и ту же степень с проверкой;

2) методу равносильных преобразований;

3) функционально-графическому методу;

4) методу введения новой переменной.

IV. Практическая часть

(Работа в группах. Каждая группа учащихся получает карточку с уравнением и решает ее в тетрадях. В это время по одному представителю от группы решают пример на доске. Учащиеся каждой группы решают тот же пример, что и член их группы, и следят за правильностью выполнения задания на доске. Если отвечающий у доски допускает ошибки, то тот, кто их замечает, поднимает руку и помогает исправить. В ходе занятия каждый учащийся помимо примера, решаемого его группой, должен записать в тетрадь и другие, предложенные группам, и решить их дома.)

Группа 1.

Группа 2.

Группа 3.

V. Самостоятельная работа

(В группах сначала идет обсуждение, а затем учащиеся приступают к выполнению задания. Правильное решение, подготовленное преподавателем, выводится на экран.)

VI. Подведение итогов урока

Теперь вы знаете, что решение иррациональных уравнений требует от вас хороших теоретических знаний, умения применять их на практике, внимания, трудолюбия, сообразительности.

Домашнее задание

Решить уравнения, предложенные группам в ходе занятия.

Иррациональные уравнения и способы их решения

1. Иррациональные уравнения и способы их решения

2009-2010 уч. год
Векслер Е. В.
ГОУ СОШ №252
Урок 1
Иррациональные
уравнения и
способы их решения

2. Какие уравнения называются ИРРАЦИОНАЛЬНЫМИ ?

Определение:
Уравнения, в которых
переменная содержится
под знаком корня, называют
иррациональными.

3. Вспомните графики функций

4. Укажите, для каких значений переменных равенство верно.

xy x y
x x
2
x x
4
2
x x 1
3
x x

5. Какие из предложенных уравнений не являются иррациональными?

6. Какие уравнения не имеют корней?

7. Какие уравнения из оставшихся можете решить?

1
3
1
2
х
7
9
2 х 7 9
3
2 х 7 729
2 х 722
х 361
Ответ : 361
11 ( х 6) 3 x 6 0
11
(
х
6
)
3
x
6
0
х 6 0
3х 6 0
х 6
Проверка
х 6 корень
Ответ : 2;6
х 2
3 6 6 0, значит
44)) хх 22хх 44 22хх 11
22
х 2х 4 2х 1
2
х 4х 3 0
2
х 3
х 1
Ответ : 1;3
3 33 х 1 3 хх 1
1010
3х 1 х 3
х
10 3 3 х 1 х
3х 1 х 3
х 3
2
х 2 9х 8 0
3
х
1
х
6х 9
х 8
х 3
х 1
2
Проверка : 3 3 8 1 8 (верно) х 9 х 8 0
х 3
3 3 1 1 1 (неверно)
Ответ : 8
х 8
х 1
3х 1 х 2 6 х 9
Ответ : 8

12.

Как предлагаете решить уравнение №9 ?

Решить уравнение, исследуя область допустимых
значений
1 х
2
4
5х 5 2

14. Как предлагаете решить уравнение № 14 ?

15. Свойства монотонности функций

f(x)=g(x)

16. Свойства монотонности функций

f(x)=a

17. Свойства монотонности функций

f(x)=a

18. Свойства монотонности функций

•Если функция y=f(x) монотонная, то
уравнение f(x)=a имеет не более одного корня.
•Если функции y=f(x) и y=g(x) имеют разный
характер монотонности, то уравнение f(x)=g(x)
имеет не более одного корня.

19. Свойства монотонности функций

•Если функции y=f(x) и y=g(x) возрастают
(убывают) на некотором множестве, то
функция y=f(x)+g(x) также возрастает
(убывает ) на этом множестве.
•Функция вида
возрастает при к>0 и убывает при к
14 1 х 2 х 3 х 0
3
3
3

21. ВНИМАНИЕ !!!!!

•Если функции различной монотонности, то
монотонность суммы, произведения, разности этих
функций определить нельзя!!
Например:
2х 3 4 х 2
2х 3 4 х 2

22.

Решить уравнение с помощью свойств монотонности.

2 х х 3 9 х

23. Как предлагаете решить уравнение № 12 ?

у х 1
3
у x 1
2
Нет корней
12 х 1 x 1
3
2

25. Какими способами теперь можем решать иррациональные уравнения?

1.Возведение в степень.
2.Уединение корня.
3.Исследование области допустимых
значений.
4.Графический способ
5.Использование свойств монотонности
х 1 х 3 2
Уединение корня
х 1 1 х
Исследование области
допустимых значений
х х 12
3
1
x 1 2
x
Графический способ
х 2х х х х
2
Возведение в степень
2
Использование
свойств
монотонности

27. Домашнее задание

§9 Иррациональные уравнения
Решить уравнения на карточке
3
1
x 1 2
x
x ≈0,9
Построим в одной и той же системе координат графики
функций:
D(y)=[0;+ ) График — кривая линия,
y(x)=
x
расположенная на промежутке [0;+ )
а)
3
g ( x)
б)
x 2
x
0
1 4 9 16
y
0
1 2 3
1
1
3
0,6
4
D(g)=(- ; — 2) (- 2; + )
Дробно-линейная функция, график – гипербола
x
y
-8 -7
— 0,5 — 0,6
-5
-1
-4
— 1,5
-3
-3
-1
3
0
1,5
4
0,5
x≈1
у 6
у х
у х2 3
у 3 х 1
у 3 х
1 3 2 х 7 9
9 х 10 3 х 2
2) 3 х 5 2
10 3 3х 1 х
3 2 х 3х 4
4) х 2 х 4 2 х 1
2
5 3 2 х 7 9
6 х 8 3 7 х
7 х х 5 1
8) х х
11 ( х 6) 3x 6 0
12 3 х 1 x 2 1
13 4 х 2 3х 2 0
14 3 1 х 3 2 х 3 3 х 0
х 1 х 3 2
Уединение корня
х 1 1 х
Исследование
области допустимых
значений
х х 12
3
1
x 1 2
x
Графический способ
х 2х х х х
2
Возведение в степень
2
Использование
свойств
монотонности

Решение иррациональных уравнений через определение корня

Примеры решения уравнений


По определению корня наиболее удобно решать простейшие иррациональные уравнения с числами в правых частях, то есть, уравнения , где C – некоторое число. Когда в правой части уравнения находится число, то даже при четном показателе корня не обязательно переходить к системе: если С – неотрицательное число, то по определению корня четной степени , а если С – отрицательное число, то сразу можно делать вывод об отсутствии корней уравнения , ведь по определению корень четной степени есть неотрицательное число, значит уравнение не обращается в верное числовое равенство ни при каких действительных значениях переменной x.



Чтобы идти от простого к сложному, с решения таких уравнений и начнем. И для начала решим простейшее иррациональное уравнение, в левой части которого находится корень четной степени, а в правой части — положительное число: , где C – положительное число. Определение корня позволяет перейти от решения заданного иррационального уравнения к решению более простого уравнения без корней С2·k=f(x).





Аналогично по определению корня решаются простейшие иррациональные уравнения с нулем в правой части.





Отдельно остановимся на иррациональных уравнениях, в левой части которых находится корень четной степени с переменной под его знаком, а в правой – отрицательное число. Такие уравнения не имеют решений на множестве действительных чисел (про комплексные корни мы будем говорить после знакомства с комплексными числами). Это довольно очевидно: корень четной степени по определению есть неотрицательное число, значит, он не может быть равен отрицательному числу.





Левые части иррациональных уравнений из предыдущих примеров были корнями четных степеней, а правые — числами. Сейчас рассмотрим примеры с переменными в правых частях, то есть, будем решать иррациональные уравнения вида . Для их решения по определению корня осуществляется переход к системе , которая имеет то же множество решений что и исходное уравнение.





Нужно иметь в виду, что систему , к решению которой сводится решение исходного иррационального уравнения , желательно решать не механически, а, по возможности, рационально. Понятно, что это больше вопрос из темы «решение систем», но все же перечислим три часто встречающихся ситуации с иллюстрирующими их примерами:



  1. К примеру, если первое ее уравнение g2·k(x)=f(x) не имеет решений, то нет смысла решать еще и неравенство g(x)≥0, ведь уже из отсутствия решений уравнения можно сделать вывод об отсутствии решений системы.



  1. Аналогично, если неравенство g(x)≥0 не имеет решений, то не обязательно решать еще и уравнение g2·k(x)=f(x), ведь и без этого понятно, что в этом случае система не имеет решений.



  1. Довольно часто неравенство g(x)≥0 вообще не решают, а лишь проверяют, какие из корней уравнения g2·k(x)=f(x) ему удовлетворяют. Множество всех тех из них, которые удовлетворяют неравенству, является решением системы, значит, является и решением равносильного ей исходного иррационального уравнения.



Достаточно про уравнения с четными показателями корней. Пора уделить внимание и иррациональным уравнениям с корнями нечетных степеней вида . Как мы уже сказали, для их решения осуществляется переход к равносильному уравнению , которое решается любыми доступными методами.





В заключение этого пункта упомянем про проверку решений. Метод решения иррациональных уравнений по определению корня гарантирует равносильность переходов. Значит, проверку найденных решений проводить не обязательно. Этот момент можно отнести к преимуществам данного метода решения иррациональных уравнений, ведь в большинстве других методов проверка является обязательным этапом решения, позволяющем отсечь посторонние корни. Но при этом следует помнить, что проверка путем подстановки найденных решений в исходное уравнение никогда не бывает лишней: вдруг где закралась вычислительная ошибка.


Нестандартные методы решения иррациональных уравнений

Нестандартные методы решения иррациональных уравнений

Куликов О. В.,Вострикова О.Ю.

МБОУ гимназия №56 11 Б класс, Тел.: (950)823 — 97 – 25, E – mail: [email protected]

1  стр. (принято к публикации)

Не всякое уравнение в результате преобразований или с помощью удачной замены переменной может быть сведено к уравнению того или иного стандартного вида, для которого существует определенный алгоритм решения. В таких случаях иногда оказывается полезным использовать другие методы решения, речь о которых и пойдет в ходе данной работы. Выше сказанное определяет актуальность исследовательской работы. Объект исследования – стандартные уравнения и уравнения, не поддающиеся решению с помощью стандартных методов, или отличающиеся громоздкостью стандартного решения. Поэтому, цель исследования состоит в том, чтобы ознакомиться с нестандартными методами решения уравнений и научится решать их этими методами, а также определить достоинства и недостатки методов решения иррациональных уравнений. Можно предположить, что применение разработанной методики решения иррациональных уравнений позволит учащимся выбирать наиболее рациональный метод для решения иррациональных уравнений, в том числе и те методы, которые не рассмотрены в школьных учебниках. Для достижения поставленной цели в данной работе решались следующие задачи: изучить статьи и учебно-методическую литературу по данной теме, подобрать теоретический материал, связанный с равносильностью уравнений равносильностью преобразований, методами решения иррациональных уравнений, а также подобрать примеры решения иррациональных уравнений для демонстрации излагаемой теории.

Практическая значимость работы состоит в том, что материалы проекта могут использоваться как справочный материал на занятиях по математике и на дополнительных курсах.

Литература

1. Колесникова С.И.. Математика. Интенсивный курс подготовки к ЕГЭ – 6 изд., 2008 год. 33 cтр.

2. Шарыгин И. Ф.. Факультативный курс по математике: Решение задач – М.: Просвещение, 1989.

3. Чулков П.В. Материалы курса «Уравнения и неравенства в школьном курсе математики»: Лекции 1-8. – М.: Педагогический университет «Первое сентября», 2006.

4. Черкасов О.Ю., Якушев А. Г.. Математика: интенсивный курс подготовки к экзамену. – М.: Айрис-пресс, 2004.

5. Ершова А.П., Голобородько В.В.. Самостоятельные и контрольные работы по алгебре и началам анализа для 10-11 классов. – М.: Илекса, 2006

6. Дьячков А.К., Иконникова Н.И., Казак В.М., Морозова Е.В.. Единый государственный экзамен. Математика. – Челябинск: Взгляд, 2006 –Ч.1,2


Уравнения иррациональные и способы их решения

Изучая алгебру, школьники сталкиваются с уравнениями многих видов. Среди тех из них, которые наиболее простые, можно назвать линейные, содержащие одну неизвестную. Если переменная в математическом выражении возводится в определенную степень, то уравнение называют квадратным, кубическим, биквадратным и так далее. Указанные выражения могут содержать рациональные числа. Но существуют также уравнения иррациональные. От прочих они отличаются наличием функции, где неизвестное находится под знаком радикала (то есть чисто внешне переменную здесь можно увидеть написанной под квадратным корнем). Решение иррациональных уравнений имеет свои характерные особенности. При вычислении значения переменной для получения правильного ответа их следует обязательно учитывать.

«Невыразимые словами»

Не секрет, что древние математики оперировали в основном рациональными числами. К таковым относятся, как известно, целые, выражаемые через обыкновенные и десятичные периодические дроби представители данного сообщества. Однако ученые Среднего и Ближнего Востока, а также Индии, развивая тригонометрию, астрономию и алгебру, иррациональные уравнения тоже учились решать. К примеру, греки знали подобные величины, но, облекая их в словесную форму, употребляли понятие «алогос», что означало «невыразимые». Несколько позднее европейцы, подражая им, называли подобные числа «глухими». От всех остальных они отличаются тем, что могут быть представлены только в форме бесконечной непериодической дроби, окончательное числовое выражение которой получить просто невозможно. Поэтому чаще подобные представители царства чисел записываются в виде цифр и знаков как некоторое выражение, находящееся под корнем второй или большей степени.

На основании вышесказанного попробуем дать определение иррациональному уравнению. Подобные выражения содержат так называемые «невыразимые числа», записанные с использованием знака квадратного корня. Они могут представлять собой всевозможные довольно сложные варианты, но в своей наипростейшей форме имеют такой вид, как на фото ниже.

Преступая к решению иррациональных уравнений, перво-наперво необходимо вычислить область допустимых значений переменной.

Имеет ли смысл выражение?

Необходимость проверки полученных значений вытекает из свойств арифметического квадратного корня. Как известно, подобное выражение приемлемо и имеет какой-либо смысл лишь при определенных условиях. В случаях корня четной степени все подкоренные выражения должны быть положительными или равняться нулю. Если данное условие не выполняется, то представленная математическая запись не может считаться осмысленной.

Приведем конкретный пример, как решать иррациональные уравнения (на фото ниже).

В данном случае очевидно, что указанные условия ни при каких значениях, принимаемых искомой величиной, выполняться не могут, так как получается, что 11 ≤ x ≤ 4. А значит, решением может являться только Ø.

Метод анализа

Из вышеописанного становится понятно, как решать иррациональные уравнение некоторых типов. Здесь действенным способом может оказаться простой анализ.

Приведем ряд примеров, которые снова наглядно это продемонстрируют (на фото ниже).

В первом случае при внимательном рассмотрении выражения сразу оказывается предельно ясно, что истинным оно быть не может. Действительно, ведь в левой части равенства должно получаться положительное число, которое никак не способно оказаться равным -1.

Во втором случае сумма двух положительных выражений может считаться равной нулю, лишь только когда х — 3 = 0 и х + 3 = 0 одновременно. А подобное опять невозможно. И значит, в ответе снова следует писать Ø.

Третий пример очень похож на уже рассмотренный ранее. Действительно, ведь здесь условия ОДЗ требуют, чтобы выполнялось следующее абсурдное неравенство: 5 ≤ х ≤ 2. А подобное уравнение аналогичным образом никак не может иметь здравых решений.

Неограниченное приближение

Природа иррационального наиболее ясно и полно может быть объяснена и познана только через нескончаемый ряд чисел десятичной дроби. А конкретным, ярким примером из членов этого семейства является πи. Не без оснований предполагается, что эта математическая константа была известна с древних времен, используясь при вычислении длин окружности и площади круга. Но среди европейцев ее впервые применили на практике англичанин Уильям Джонс и швейцарец Леонард Эйлер.

Возникает эта константа следующим образом. Если сравнивать самые разные по длине окружности, то отношение их длин и диаметров в обязательном порядке равны одному и тому же числу. Это и есть πи. Если выразить его через обыкновенную дробь, то приблизительно получим 22/7. Впервые это сделал великий Архимед, портрет которого представлен на рисунке выше. Именно поэтому подобное число получило его имя. Но это не явное, а приближенное значение едва ли не самого удивительного из чисел. Гениальный ученый с точностью до 0,02 нашел искомую величину, но, по сути, данная константа не имеет реального значения, а выражается как 3,1415926535… Она представляет собой бесконечный ряд цифр, неограниченно приближаясь к некоему мифическому значению.

Возведение в квадрат

Но вернемся к иррациональным уравнениям. Чтобы отыскать неизвестное, в данном случае очень часто прибегают к простому методу: возводят обе части имеющегося равенства в квадрат. Подобный способ обычно дает хорошие результаты. Но следует учитывать коварство иррациональных величин. Все полученные в результате этого корни необходимо проверять, ведь они могут не подойти.

Но продолжим рассмотрение примеров и постараемся найти переменные вновь предложенным способом.

Совсем несложно, применив теорему Виета, найти искомые значения величин после того, как в результате определенных оперций у нас образовалось квадратное уравнение. Здесь получается, что среди корней будут 2 и -19. Однако при проверке, подставив полученные значение в изначальное выражение, можно убедиться, что ни один из этих корней не подходит. Это частое явление в иррациональных уравнениях. Значит, наша дилемма вновь не имеет решений, а в ответе следует указать пустое множество.

Примеры посложней

В некоторых случаях требуется возводить в квадрат обе части выражения не один, а несколько раз. Рассмотрим примеры, где требуется указанное. Их можно увидеть ниже.

Получив корни, не забываем их проверять, ведь могут возникнуть лишние. Следует пояснить, почему такое возможно. При применении подобного метода происходит в некотором роде рационализация уравнения. Но избавляясь от неугодных нам корней, которые мешают производить арифметические действия, мы как бы расширяем существующую область значений, что чревато (как можно понять) последствиями. Предвидя подобное, мы и производим проверку. В данном случае есть шанс убедиться, что подходит только один из корней: х = 0.

Системы

Что же делать в случаях, когда требуется осуществить решение систем иррациональных уравнений, и у нас в наличии не одно, а целых два неизвестных? Здесь поступаем так же, как в обычных случаях, но с учетом вышеперечисленных свойств данных математических выражений. И в каждой новой задаче, разумеется, следует применять творческий подход. Но, опять же, лучше рассмотреть все на конкретном примере, представленном ниже. Здесь не просто требуется найти переменные х и у, но и указать в ответе их сумму. Итак, имеется система, содержащая иррациональные величины (см. фото ниже).

Как можно убедиться, подобная задача не представляет ничего сверхъестественно сложного. Требуется лишь проявить сообразительность и догадаться, что левая часть первого уравнения представляет собой квадрат суммы. Подобные задания встречаются в ЕГЭ.

Иррациональное в математике

Каждый раз потребность в создании новых видов чисел возникала у человечества тогда, когда ему не хватало «простора» для решения каких-то уравнений. Иррациональные числа не являются исключением. Как свидетельствуют факты из истории, впервые великие мудрецы обратили на это внимание еще до нашей эры, веке в VII. Сделал это математик из Индии, известный под именем Манава. Он отчетливо понимал, что из некоторых натуральных чисел невозможно извлечь корень. К примеру, к таковым относятся 2; 17 или 61, а также многие другие.

Один из пифагорейцев, мыслитель по имени Гиппас, пришел к тому же выводу, пытаясь производить вычисления с числовыми выражениями сторон пентаграммы. Открыв математические элементы, которые не могут быть выражены цифровыми значениями и не обладают свойствами обычных чисел, он настолько разозлил своих коллег, что был выброшен за борт корабля, в море. Дело в том, что другие пифагорейцы сочли его рассуждения бунтом против законов вселенной.

Знак радикала: эволюция

Знак корня для выражения числового значения «глухих» чисел стал использоваться при решении иррациональных неравенств и уравнений далеко не сразу. Впервые о радикале начали задумываться европейские, в частности итальянские, математики приблизительно в XIII веке. Тогда же для обозначения придумали задействовать латинскую R. Но немецкие математики в своих работах поступали иначе. Им больше понравилась буква V. В германии вскоре распространилось обозначение V(2), V(3), что призвано было выражать корень квадратный из 2, 3 и так далее. Позднее в дело вмешались нидерландцы и видоизменили знак радикала. А завершил эволюцию Рене Декарт, доведя знак квадратного корня до современного совершенства.

Избавление от иррационального

Иррациональные уравнения и неравенства могут включать в себя переменную не только под знаком квадратного корня. Он может быть любой степени. Самым распространенным способом от него избавиться является возможность возвести обе части равенства в соответствующую степень. Это основное действие, помогающее при операциях с иррациональным. Действия в четных случаях особенно не отличаются от тех, которые были уже разобраны нами ранее. Здесь должны быть учтены условия неотрицательности подкоренного выражения, а также по окончании решения необходимо производить отсев посторонних значений переменных таким образом, как было показано в рассмотренных уже примерах.

Из дополнительных преобразований, помогающих найти правильный ответ, часто используется умножение выражения на сопряженное, а также нередко требуется введение новой переменной, что облегчает решение. В некоторых случаях, чтобы отыскать значение неизвестных, целесообразно применять графики.

Методы решения иррациональных уравнений цели

Шибанова Татьяна Павловна


Методы
решения иррациональных уравнений.

Цели:

  • Образовательная
    –познакомить учащихся с нестандартными
    методами решения иррациональных
    уравнений; систематизировать знания
    учащихся о методах решения иррациональных
    уравнений, способствовать формированию
    умений классифицировать иррациональные
    уравнения по методам решений, научить
    применять эти методы, выбирать
    рациональный путь решения.

  • Развивающая
    –способствовать
    развитию математического кругозора,
    логического мышления.

  • Воспитательная
    – содействовать
    воспитанию интереса к иррациональным
    уравнениям, воспитывать чувство
    коллективизма, самоконтроля,
    ответственности.

Задачи урока:

  1. Повторить
    определение и основные методы решения
    иррациональных уравнений;

  2. Продемонстрировать
    нестандартные методы решения
    иррациональных уравнений; формировать
    умение выбирать рациональные пути
    решения;

  3. Освоение всеми
    учащимися алгоритмов решения
    иррациональных уравнений, закрепление
    теоретических знаний при решении
    конкретных примеров;

  4. Развитие у учащихся
    логического мышления в процессе поиска
    рациональных методов и алгоритмов
    решения;

  5. Развитие культуры
    научных и учебных взаимоотношений
    между учениками и между учениками и
    учителем; воспитание навыков совместного
    решения задач.

Методы
обучения
:

  • Информационно-
    иллюстративный;

  • репродуктивный;

  • проблемный
    диалог;

  • частично-поисковый;

  • системные
    обобщения.

Формы
организации учебной деятельности:

  • Фронтальная,

  • групповая,

  • самопроверка,

  • взаимопроверка,

  • коллективные
    способы обучения.

Оборудование
урока:
компьютер,
проектор, карточки с заданием, лист
учета знаний.

Продолжительность
занятия
: 2 урока
по 45 минут.


План
урока:

  1. Организационный
    момент. Постановка цели, мотивация.

  2. Актуализация
    опорных знаний, проверка домашней
    работы.

  3. Изучение
    нового материала.

  4. Закрепление
    изученного материала на данном уроке
    и ранее пройденного, связанного с новым.

  5. Подведение
    итогов и результатов урока. Рефлексия.

  6. Задание
    на дом.

Конспект
урока.

  1. Организационный
    момент. Постановка цели, мотивация.

  2. Актуализация
    опорных знаний

    проводится в форме беседы по лекционному
    материалу по данной теме с использованием
    компьютерной презентации. Проверка
    домашнего задания.

Уравнение, содержащее
переменные под знаком корня или дробной
степени, называется
иррациональным.

Назовите
иррациональные уравнения:

Это значит
найти все такие значения переменной,
при которых уравнение превращается в
верное равенство, либо доказать, что
таких значений не существует.

  1. Уединение
    радикала. Возведение в степень.

a)
При решении иррационального уравнения
с радикалом четной
степени

возможны два
пути
:

  1. использование
    равносильных
    преобразований

для уравнения
вида

для уравнения
вида

  1. после возведения в
    степень выполнение
    проверки
    , так
    как возможно появление посторонних
    корней

b)
При
решении иррационального уравнения с
радикалом нечетной
степени

возведение в нечетную степень правой
и левой части уравнения всегда приводит
к равносильному уравнению и потеря
корней или их приобретения происходить
не может.

Пример
1:

Ответ:
x=1

Пример
2:

Ответ:
x=1

Пример
3:

Проверка: x=2

x=5


посторонний корень

Ответ:
x=2

Если
радикалов несколько, то уравнение
возводить в степень приходится возводить
неоднократно.

Пример
4:

Проверка
показывает, что оба корня подходят.

Ответ:

  1. Метод введения
    вспомогательного неизвестного или
    “метод замены

Пример
5:

Сделаем
замену

причём

тогда

не
удовлетворяет условию

Возвращаемся
к замене:



Проверка показывает,
что оба корня подходят.

Ответ:1;2

Иногда
удобно ввести не одну, а несколько
переменных.

Пример
6
:

.

Заметим, что знаки х под радикалом
различные. Введем обозначение

,
.

Тогда,

Выполним почленное сложение обеих
частей уравнения
.

Имеем систему уравнений

Т.к. а + в = 4, то

Значит:

9 – x = 8 , х = 1.

Ответ : х = 1

            1. Метод разложения
              на множители или расщепления.

    • Произведение равно
      нулю тогда и только тогда, когда хотя
      бы один из входящих в него сомножителей
      равен нулю, а остальные при этом имеют
      смысл.

    Пример 7:

    Ответ:
    -4;3

    1. Изучение
      нового материала.

    Нестандартные
    методы решения иррациональных уравнений.

    1. Умножение
      на сопряжённое выражение.

    2. Переход
      к модулю.

    3. Использование
      свойств функции:

        • Область
          определения функции (ОДЗ)

        • Область
          значения функции

        • Свойство
          ограниченности функции (метод оценок)

        • Свойство
          монотонности

        • Использование
          суперпозиций функций

    Воспользуемся
    формулой

    Пример 8:

    Умножим
    обе части уравнения на сопряжённое
    выражение:

    Проверка
    показывает, что число является корнем.

    Ответ:

    Для этого
    метода воспользуемся тождеством:

    Пример
    9:

    Рассмотрим случаи:

    тогда

    2=6( ложно)

    Ответ: -3;3

    Иногда
    нахождение области определения функций,
    входящих в уравнение, существенно
    облегчает его решение.

    Пример
    10:

    ОДЗ:

    ОДЗ:
    x=0
    и x=1

    Проверка
    показывает, что только x=1
    является корнем.

    Ответ:

    Пример
    11:

    ,
    тогда

    Тогда

    невозможно.

    Ответ:
    корней нет.

    Пример
    12:

    Данное
    уравнение не имеет решений, так как его
    левая часть- функция

    может принимать только неотрицательные
    значения.

    Ответ:
    корней нет

    Пример
    13:

    Учитывая
    то, что левая часть уравнения – функция

    может принимать только неотрицательные
    значения, решим неравенство:

    неравенство решений
    не имеет, тогда и исходное уравнение
    тоже.

    Ответ:
    корней нет

    Пример 14:

    Заметим,
    что
    ,
    т.е.
    ,
    а

    Проверка
    показывает, что это значение является
    и корнем второго уравнения.

    Ответ:

    Пример 15:
    .

    Рассмотрим
    функции

    и

    .

    монотонно возрастает,
    а

    — убывает, следовательно, уравнение
    имеет не более одного корня.

    Значение
    корня легко найти подбором:

    Ответ:

    Пример 16:

    Функция

    возрастает на своей области определения,
    как сумма двух возрастающих функций,
    следовательно, уравнение

    имеет не более одного корня. Так как
    ,
    то

    — единственный корень .

    Ответ:

    Пример 17:

    Запишем
    уравнение в виде

    Рассмотрим
    функцию

    — монотонно возрастающую, тогда уравнение
    имеет вид
    .
    Оно равносильно уравнению

    Сделаем
    замену

    не удовлетворяет
    условию

    Ответ:

    1. Закрепление
      изученного материала

      на данном уроке и ранее пройденного,
      связанного с новым.

    Решение уравнений в
    группах по 6 человек.

    Ребята получают карточку
    с заданием. Решение уравнений обсуждают
    вместе, записывают его.

    После выполнения
    группами заданий проводится взаимопроверка.
    Группы меняются заданиями с решениями
    по кругу:

    1
    6 5

    2
    3 4

    Учащиеся групп обсуждают
    решение, исправляют ошибки и выставляют
    оценки.

    Потом работы с
    выставленными оценками возвращаются
    в группы для обсуждения вклада каждого
    в решение проблемы.

    Выставляются каждому
    оценки с занесением в оценочную таблицу.
    Учитель контролирует и вносит, если
    нужно, свои коррективы.

    1. Подведение
      итогов и результатов урока. Рефлексия.

    2. Задание
      на дом:

    Решить
    уравнения:

    1. *

    Используемая
    литература.

    1. Чулков П.В.
      Материалы курса «Уравнения и неравенства
      в школьном курсе математики»: Лекции
      1-8. – М.: Педагогический университет
      «Первое сентября», 2006.

    2. Дьячков А.К.,
      Иконникова Н.И., Казак В.М., Морозова
      Е.В. Единый государственный экзамен.
      Математика. – Челябинск: Взгляд, 2006
      –Ч.1,2

    3. Шарыгин И. Ф.
      Факультативный курс по математике:
      Решение задач. – М.: Просвещение, 1989

    4. Черкасов О.Ю.,
      Якушев А.Г. Математика: интенсивный
      курс подготовки к экзамену. – М.:
      Айрис-пресс, 2004.

    5. Ершова А.П.,
      Голобородько В.В. Самостоятельные и
      контрольные работы по алгебре и началам
      анализа для 10-11 классов. – М.: Илекса,
      2006.

    Задания
    для работы в группах:

    Вариант 1(1,3,5
    группы).

    Решите уравнения,

    используя подсказку:

    1. Возведи обе части в
      квадрат:

    1. Выполни замену:

    1. Найди ОДЗ:

    1. Умножай на сопряжённое
      выражение:

    1. Переходи к модулю:

    1. Используй свойства
      функций:

    1. Реши любым способом:

    Вариант
    2
    (
    2,4,6 группы)

    Решите уравнения,

    используя подсказку:

    1. Возведи обе части в
      квадрат:

    1. Выполни замену:

    1. Найди ОДЗ:

    1. Умножай на сопряжённое
      выражение:

    1. Переходи к модулю:

    1. Используй свойства
      функций:

    1. Реши любым способом:

    Проверочная
    работа
    по теме:
    «Методы

    Вариант 1

    Решите уравнения,

    используя подсказку:

    1. Возведи обе части в
      квадрат:

    1. Выполни замену:

    1. Найди ОДЗ:

    1. Разложи на множители:

    1. Умножай на сопряжённое
      выражение:

    1. Переходи к модулю:

    1. Используй свойства
      функций:

    1. Реши любым способом:

    решения иррациональных
    уравнений
    »

    Вариант 2

    Решите уравнения,

    используя подсказку:

    1. Возведи обе части в
      квадрат:

    1. Выполни замену:

    1. Найди ОДЗ:

    1. Разложи на множители:

    1. Умножай на сопряжённое
      выражение:

    1. Переходи к модулю:

    1. Используй свойства
      функций:

    1. Реши любым способом:

    College Algebra
    Урок 19: Радикальные уравнения и Уравнения с рациональными показателями


    WTAMU > Виртуальная математическая лаборатория> Алгебра колледжа

    Цели обучения


    По завершении этого руководства вы сможете:

    1. Решите радикальные уравнения.
    2. Решите уравнения с рациональными показателями.

    Введение


    В этом уроке мы рассмотрим решение двух разных типов
    уравнений, радикальных уравнений и уравнений с рациональными показателями.
    Оба эти уравнения преследуют одну и ту же конечную цель — получить вашу переменную
    с одной стороны, а все остальное — с другой, используя обратные операции.
    Кроме того, после удаления радикального или рационального показателя в уравнениях
    в этом уроке они превращаются в линейное или квадратное уравнение.
    Хорошие новости и плохие новости, как упоминалось в других уроках, много
    раз в математике вы используете предыдущие знания, чтобы помочь выработать новые концепции.
    Это хорошо, потому что вам не нужно подходить к проблеме полностью.
    новый и узнайте все новые шаги. Это может быть ошеломляющим.это
    плохо, потому что нужно помнить вещи из прошлого. Иногда
    мы заставляем себя истощить наши мозги после прохождения теста, а иногда
    забыть то, что мы узнали. Если вам нужен обзор радикалов в
    в общем, не стесняйтесь переходить к Tutorial
    4: радикалы
    . Если вам нужен обзор рациональных показателей в
    в общем, не стесняйтесь переходить к Tutorial
    5. Рациональные экспоненты
    .Если вам нужен обзор решения линейных
    уравнения, не стесняйтесь переходить к учебному пособию
    14. Линейные уравнения с одной переменной
    . Если вам нужен обзор
    по решению квадратных уравнений см. Учебное пособие
    17. Квадратные уравнения
    . Пройдя по этой странице,
    вы должны быть опытным профессионалом в работе с корнями. Я думаю ты готов
    решить эти уравнения.

    Учебник


    Решение радикальных уравнений

    Шаг 1 : Изолировать
    один из радикалов.

    Другими словами, получите один радикал с одной стороны, а все остальное — с другой.
    другие с использованием обратных операций.

    В некоторых проблемах всего один радикал. Однако есть
    некоторые проблемы, у которых более одного радикала. В этих проблемах
    убедитесь, что вы изолировали только один.

    Шаг 2 : Избавиться
    вашего радикального знака.

    Операция, обратная радикалу или корню, — это возвести его в степень.
    Какой показатель? Хороший вопрос, это будет показатель степени, соответствующий
    индекс или корень вашего корня.

    Другими словами, если бы у вас был квадратный корень, вам пришлось бы возвести его в квадрат.
    чтобы избавиться от этого. Если бы у вас был кубический корень, вам пришлось бы куб
    чтобы избавиться от него и так далее.

    Вы можете возвести обе стороны во 2-ю степень, 10-ю степень, сотую степень,
    и т. д. Если вы проделаете то же самое с обеими сторонами уравнения,
    обе стороны останутся равными друг другу.

    Шаг 3 : Если вы
    остался радикальный знак, повторите шаги 1 и 2.

    Иногда вы начинаете с двух или более радикалов в своем уравнении.
    Если это так и у вас есть хотя бы один нерадикальный термин, вы
    наверное придется повторить шаги 1 и 2.

    Шаг 4 : Решите
    оставшееся уравнение.

    Шаг 5 : Проверить
    для посторонних решений.

    При решении радикальных уравнений могут появиться дополнительные решения, когда вы
    поднимите обе стороны до равной мощности. Эти дополнительные решения называются
    посторонние решения. Если значение является посторонним
    решение, это не решение исходной проблемы.

    В радикальных уравнениях вы проверяете наличие посторонних решений, вставляя
    в значениях, которые вы нашли обратно в исходную проблему.Если левая сторона
    не совпадает с правой стороной, значит у вас постороннее решение.

    Радикал в этом уравнении уже выделен.

    Если возвести квадратный корень, он исчезнет. Это то, что
    мы хотим сделать здесь так, чтобы мы могли получить x
    из-под квадратного корня и продолжайте решать его.

    * Возведение квадратного корня обратное.
    это

    Шаг 3 : Если вы
    все еще есть радикальный левый, повторите шаги 1 и 2.

    Радикалов больше нет, поэтому нам не нужно повторять шаги 1 и 2.

    Шаг 4 : Решить
    оставшееся уравнение.

    * Инверсия доп. 5 является суб. 5

    * Инверсная по отношению к мульт. на 2 — div. по 2

    Давайте проверим, не является ли x = 22 посторонним
    раствор:

    * Вставка 22 для x

    * Истинное утверждение

    Поскольку мы получили истинное утверждение, x = 22
    не является посторонним решением.

    Есть одно решение этого радикального уравнения: x = 22.

    * Инверсия доп. x — это под. х

    * Квадратный корень находится на одной стороне уравнения.

    Если возвести квадратный корень, он исчезнет.Это то, что
    мы хотим сделать здесь так, чтобы мы могли получить x
    из-под квадратного корня и продолжайте решать его.

    Будьте осторожны с этим. Это
    ОЧЕНЬ ИСКУССТВО возвести в квадрат левую часть члена за членом и получить 9 — x в квадрате или 9 + x в квадрате. Однако вы
    нужно выровнять его как сторону, как показано выше. Вспомните, когда вы
    возвести бином в квадрат, вы получите первый член в квадрате минус удвоенное произведение
    из двух членов плюс второй член в квадрате.Если вам нужен обзор
    по возведению бинома в квадрат см. Учебное пособие
    6. Полиномы
    .

    Шаг 3 : Если вы
    все еще есть радикальный левый, повторите шаги 1 и 2.

    Радикалов больше нет, поэтому нам не нужно повторять шаги 1 и 2.

    Шаг 4 : Решить
    оставшееся уравнение.

    В этом примере уравнение, полученное в результате возведения в квадрат обеих сторон
    оказалось квадратным уравнением .

    Если вам нужен обзор решения квадратных уравнений, смело переходите
    по Урок 17: Квадратичные уравнения.

    Давайте проверим, не является ли x = 6 посторонним
    раствор:

    * Вставка 6 для x

    * Ложная выписка

    Поскольку мы получили ложное утверждение, x =
    6 — постороннее решение.

    Давайте проверим, является ли x = 2 посторонним
    раствор:

    * Вставка 2 для x

    * Истинное утверждение

    Поскольку мы получили истинное утверждение, x = 2 равно
    решение.

    Есть только одно решение этого радикального уравнения: x = 2.

    * Обратное от sub. sq. корень добавлен. кв.
    корень

    * Один квадратный корень находится на одной стороне
    экв.

    Если возвести квадратный корень, он исчезнет.Это то, что
    мы хотим сделать здесь так, чтобы мы могли получить y из
    из-под квадратного корня и продолжайте решать его.

    * Возведение квадратного корня обратно в квадрат.
    it

    * Правая сторона — двучлен
    в квадрате

    Будьте осторожны с этим. Это
    ОЧЕНЬ заманчиво возвести в квадрат правую часть члена за членом и получить 1
    + ( y + 1). Однако вам нужно возвести в квадрат
    это как сторона, как показано выше. Вспомните, что когда вы возводите двучлен в квадрат
    вы получаете квадрат первого члена плюс удвоенное произведение двух членов
    плюс второй член в квадрате. Если вам нужен обзор квадратов
    биномиальный, не стесняйтесь перейти к Учебник
    6. Полиномы
    .

    Шаг 3 : Если вы
    все еще есть радикальный левый, повторите шаги 1 и 2.

    * Инверсия доп. y и 2 суб. и и 2

    * Квадратный корень находится на одной стороне уравнения.

    * Возведение квадратного корня обратно пропорционально
    it

    * Левая сторона — двучлен
    в квадрате

    Будьте осторожны с этим. Это
    ОЧЕНЬ заманчиво возвести в квадрат левую часть члена за член и получить y в квадрате
    плюс 1. Однако вам нужно выровнять его как сторону, как показано выше.
    Вспомните, что когда вы возводите двучлен в квадрат, вы получаете первый член в квадрате плюс
    удвоенное произведение двух членов плюс второй квадрат в квадрате.
    Если вам нужен обзор возведения бинома в квадрат, см. Учебное пособие .
    6. Полиномы
    .

    Шаг 4 : Решить
    оставшееся уравнение.

    В этом примере уравнение, полученное в результате возведения в квадрат обеих сторон
    оказалось квадратным уравнением .

    Если вам нужен обзор решения квадратных уравнений, смело переходите
    по Урок 17: Квадратичные уравнения.

    Давайте проверим, не является ли y = 3 посторонним
    раствор:

    * Вставка 3 для y

    * Истинное утверждение

    Поскольку мы получили истинное утверждение, y = 3
    это решение.

    Давайте проверим, не является ли y = -1 посторонним
    раствор:

    * Вставка -1 для y

    * Истинное утверждение

    Поскольку мы получили истинное утверждение, y = -1
    это решение.

    У этого радикального уравнения есть два решения: y = 3 и y = -1.

    Решение уравнений с
    Рациональная экспонента

    И можно записать в виде

    где B — любое действительное число.

    Шаг 1 : Изолировать
    основание с рациональным показателем.

    Другими словами, получить основание с рациональным показателем на одной стороне
    а все остальное — с помощью обратных операций.

    Шаг 2 : Избавиться
    рационального показателя.

    Операция, обратная рациональному показателю, — это возвести его в
    величина, обратная этой экспоненте.
    Это позволит избавиться от рациональной экспоненты в выражении.

    Например, если рациональный показатель равен 2/3, то обратная операция
    состоит в том, чтобы возвести обе стороны в степень 3/2.

    Вы можете поднять обе стороны на ЛЮБУЮ мощность.Пока ты делаешь то же самое
    что к обеим сторонам уравнения, две стороны останутся равными
    друг с другом.

    Шаг 3 : Решите
    оставшееся уравнение.

    Шаг 4 : Проверить
    для посторонних решений.

    При решении уравнений с рациональными показателями дополнительные решения могут
    возникает, когда вы поднимаете обе стороны до равной мощности. Эти дополнительные решения
    называются посторонними решениями. Если значение
    это постороннее решение, это не решение исходной проблемы.

    В уравнениях с рациональными показателями вы проверяете наличие посторонних решений.
    вставив найденные вами значения обратно в исходную проблему.Если
    левая сторона не равна правой стороне, чем у вас есть посторонний
    решение.

    База с рациональной экспонентой уже изолирована.

    Если вы возводите выражение с рациональным показателем в обратную
    этого рационального показателя экспонента исчезнет.Этот
    это то, что мы хотим сделать здесь, чтобы мы могли получить x из-под рациональной экспоненты и продолжить его решение.

    * Обратно к степени 3/2
    это

    доводя его до степени 2/3

    * Использовать деф.
    крысы. ехр

    Шаг 3 : Решить
    оставшееся уравнение.

    В этом примере уравнение, полученное в результате увеличения обеих сторон
    в 2/3 степени оказалось линейным уравнением .

    Если вам нужен обзор решения линейных уравнений, перейдите к Tutorial
    14. Линейные уравнения с одной переменной
    .

    * Инверсия доп.1 является суб. 1

    * Инверсная по отношению к мульт. на 3 — div. по 3

    Давайте проверим, не является ли x = 5 посторонним
    раствор:

    * Вставка 5 для x

    * Истинное утверждение

    Поскольку мы получили истинное утверждение, x = 5
    это решение.

    Есть одно решение этого уравнения рациональной экспоненты: x = 5.

    * Обратное от sub. 10 — доп. 10

    * Инверсная по отношению к мульт. на 2 — div. по 2

    * крыса.опыт выражение само по себе на одной стороне
    экв.

    Если вы возводите выражение с рациональным показателем в обратную
    этого рационального показателя экспонента исчезнет. Этот
    это то, что мы хотим сделать здесь, чтобы мы могли получить x из-под рациональной экспоненты и продолжить его решение.

    * Обратно к 5/3 степени
    равно
    в степени 3/5

    Шаг 3 : Решить
    оставшееся уравнение.

    В этом примере уравнение, полученное в результате повышения обеих сторон
    в степени 3/5 оказалось линейным уравнением .

    Также обратите внимание, что это уже решено для x .
    Итак, для этого примера нам не нужно ничего делать на этом шаге.

    Проверим, нет ли постороннего решения:

    * Подключение 5 к 3/5 мощности для x

    * Истинное заявление

    Поскольку мы получили верное утверждение, это решение.

    Есть одно решение этого уравнения рациональной экспоненты:.

    * Инверсия доп. 1 является суб. 1

    * крыса. опыт выражение само по себе на одной стороне
    экв.

    Если вы возводите выражение с рациональным показателем в обратную
    этого рационального показателя экспонента исчезнет.Этот
    это то, что мы хотим сделать здесь, чтобы мы могли получить x из-под рациональной экспоненты и продолжить его решение.

    * Обратное преобразование в степень 5/2
    это

    в степени 2/5
    * Корень 5-й степени из -1 равен -1, а -1 в квадрате равен
    1.

    Шаг 3 : Решить
    оставшееся уравнение.

    В этом примере уравнение, полученное в результате возведения в квадрат обеих сторон
    оказалось квадратным уравнением .

    Если вам нужен обзор решения квадратных уравнений, смело переходите
    по Урок 17: Квадратичные уравнения.

    Давайте проверим, является ли x = -4 посторонним
    раствор:

    * Вставка -4 для x

    * Ложная выписка

    Поскольку мы получили ложное утверждение, x =
    -4 — постороннее решение.

    Давайте проверим, не является ли x = -1 посторонним
    раствор:

    * Вставка -1 для x

    * Ложная выписка

    Поскольку мы получили ложное утверждение, x =
    -1 — постороннее решение.

    Это уравнение рациональной экспоненты не имеет решения.

    Практические задачи


    Это практические задачи, которые помогут вам перейти на следующий уровень.
    Это позволит вам проверить и понять, понимаете ли вы эти
    типы проблем. Math работает как и все
    в противном случае, если вы хотите добиться успеха в этом, вам нужно практиковать это.
    Даже лучшие спортсмены и музыканты получали помощь и много
    практиковаться, практиковаться, практиковаться, чтобы стать лучше в своем виде спорта или инструменте.

    На самом деле не бывает слишком много практики.

    Чтобы получить максимальную отдачу от них, вы должны решить проблему с .
    свой, а затем проверьте свой ответ, щелкнув ссылку для ответа / обсуждения
    для этой проблемы
    . По ссылке вы найдете ответ
    а также любые шаги, которые позволили найти этот ответ.

    Практика
    Задачи 1a — 1b:
    Решите каждое радикальное уравнение.

    Практика
    Задачи 2a — 2b:
    Решите каждое уравнение рациональной экспоненты.

    Нужна дополнительная помощь по этим темам?



    WTAMU > Виртуальная математическая лаборатория> Алгебра колледжа

    Видео на этом сайте были созданы и продюсированы Ким Сьюард и Вирджиния Уильямс Трайс.
    Последний раз редактировал Ким Сьюард 16 декабря 2009 г.
    Авторские права на все содержимое (C) 2002 — 2010, WTAMU и Kim Seward.
    Все права защищены.

    Решение радикальных уравнений — промежуточная алгебра

    Цели обучения

    К концу этого раздела вы сможете:

    • Решите радикальные уравнения
    • Решите радикальные уравнения с двумя радикалами
    • Использование радикалов в приложениях

    Прежде чем начать, пройдите тест на готовность.

    1. Упростить:

      Если вы пропустили эту проблему, просмотрите (рисунок).

    2. Решить:

      Если вы пропустили эту проблему, просмотрите (рисунок).

    3. Решить

      Если вы пропустили эту проблему, просмотрите (рисунок).

    Решите радикальные уравнения

    В этом разделе мы решим уравнения, у которых есть переменная в подкоренном выражении радикального выражения. Уравнение этого типа называется радикальным уравнением.

    Радикальное уравнение

    Уравнение, в котором переменная находится в подкоренном выражении радикального выражения, называется радикальным уравнением .

    Как обычно, при решении этих уравнений, то, что мы делаем с одной стороной уравнения, мы должны делать и с другой стороной. Как только мы изолировали радикал, наша стратегия будет заключаться в возведении обеих сторон уравнения в степень индекса. Это устранит радикал.

    Решение радикальных уравнений, содержащих четный индекс, путем возведения обеих частей в степень индекса может привести к алгебраическому решению, которое не было бы решением исходного радикального уравнения. Опять же, мы называем это посторонним решением, как и при решении рациональных уравнений.

    В следующем примере мы увидим, как решить радикальное уравнение. Наша стратегия основана на повышении радикала с индексом n до n th степени. Это устранит радикал.

    Как решить радикальное уравнение

    Решить:

    Решить:

    Решить:

    Решите радикальное уравнение с одним радикалом.

    1. Выделите радикал на одной стороне уравнения.
    2. Возвести обе части уравнения в степень индекса.
    3. Решите новое уравнение.
    4. Проверьте ответ в исходном уравнении.

    Когда мы используем знак корня, он указывает на главный или положительный корень. Если в уравнении есть радикал с четным индексом, равным отрицательному числу, это уравнение не будет иметь решения.

    Решить:

    The resulting equation is square root of the quantity 9 k minus 2 in parentheses plus 1 minus 1 equals 0 minus 1. Simplifying this we get square root of the quantity 9 k minus 2 in parentheses equals negative 1. Since the square root of a real number is always positive there is no solution to the equation.» data-label=»»>

    Чтобы выделить радикал, вычтите 1 с обеих сторон.
    Упростить.

    Поскольку квадратный корень равен отрицательному числу, уравнение не имеет решения.

    Решить:

    Решить:

    Если одна сторона уравнения с квадратным корнем является биномом, мы используем образец произведения биномиальных квадратов, когда возводим его в квадрат.

    Биномиальные квадраты

    Не забудьте про средний срок!

    Решить:

    Решить:

    Решить:

    При индексе радикала 3 кубиком с обеих сторон убираем радикал.

    Решить:

    Решить:

    Решить:

    Иногда уравнение может содержать рациональные показатели вместо радикала. Мы используем те же методы для решения уравнения, что и в случае радикала. Возводим каждую часть уравнения в степень знаменателя рациональной экспоненты. Так как у нас, например,

    Помните и

    Решить:

    Решить:

    Решить:

    Иногда решение радикального уравнения приводит к двум алгебраическим решениям, но одно из них может быть посторонним решением!

    Решить:

    Решить:

    Решить:

    Когда перед радикалом стоит коэффициент, мы также должны возвести его в степень индекса.

    Решить:

    Решить:

    Решить:

    Решите радикальные уравнения с двумя радикалами

    Если в радикальном уравнении есть два радикала, мы начинаем с выделения одного из них. Часто бывает проще сначала выделить более сложный радикал.

    В следующем примере, когда один радикал изолирован, второй радикал также изолирован.

    Решить:

    Решить:

    Решить:

    Иногда после возведения обеих частей уравнения в степень у нас все еще остается переменная внутри радикала.Когда это произойдет, мы повторяем шаги 1 и 2 нашей процедуры. Выделяем радикал и снова возводим обе части уравнения в степень индекса.

    Как решить радикальное уравнение

    Решить:

    Решить:

    Решить:

    Мы резюмируем шаги здесь. Мы скорректировали наши предыдущие шаги, чтобы включить в уравнение более одного радикала. Теперь эта процедура будет работать для любых радикальных уравнений.

    Решите радикальное уравнение.

    1. Выделите один из радикальных членов на одной стороне уравнения.
    2. Возвести обе части уравнения в степень индекса.
    3. Есть еще радикалы?

      Если да, повторите шаги 1 и 2 еще раз.

      Если нет, решите новое уравнение.

    4. Проверьте ответ в исходном уравнении.

    Будьте осторожны, возводя двучлены в квадрат в следующем примере. Помните, что узор или

    Решить:

    Решить:

    Решить:

    Использование радикалов в приложениях

    По мере прохождения курсов в колледже вы будете сталкиваться с формулами, которые включают радикалы во многих дисциплинах.Мы немного изменим нашу стратегию решения проблем для геометрических приложений, чтобы дать нам план решения приложений с формулами из любой дисциплины.

    Используйте стратегию решения проблем для приложений с формулами.

    1. Прочтите задачу и убедитесь, что все слова и идеи понятны. При необходимости нарисуйте фигуру и пометьте ее данной информацией.
    2. Определите то, что мы ищем.
    3. Назовите то, что мы ищем, выбрав переменную для его представления.
    4. Переведите в уравнение, написав соответствующую формулу или модель для ситуации. Подставьте в данную информацию.
    5. Решите уравнение , используя хорошие методы алгебры.
    6. Отметьте ответ в проблеме и убедитесь, что он имеет смысл.
    7. Ответьте на вопрос полным предложением.

    Одно из применений радикалов связано с действием силы тяжести на падающие предметы.Формула позволяет определить, сколько времени потребуется упавшему предмету, чтобы ударить его по земле.

    Падающие предметы

    На Земле, если объект падает с высоты х футов, время в секундах, которое потребуется, чтобы достичь земли, определяется по формуле

    Например, если объект падает с высоты 64 фута, мы можем вычислить время, необходимое для достижения земли, подставив его в формулу.

    Извлеките квадратный корень из 64.
    Упростим дробь.

    Чтобы объект, упавший с высоты 64 фута, достиг земли, потребуется 2 секунды.

    Марисса сбросила солнцезащитные очки с моста на высоте 400 футов над рекой. Используйте формулу, чтобы определить, сколько секунд потребовалось солнцезащитным очкам, чтобы добраться до реки.

    So t equals 20 divided by 4. So t equals 5. Step 6 is to check the answer in the problem and make sure it makes sense. Does 5 equal the square root of 400 divided 4. Since 5 equals 20 divided by 4, the answer is a solution to the equation. Does 5 seconds seem like a reasonable length of time? Yes. Step 7 is to answer the question. It will take 5 seconds for the sunglasses to reach the river.» data-label=»»>

    Шаг 1. Прочтите , в чем проблема.
    Шаг 2.Определите , что мы ищем. время, необходимое для

    солнцезащитные очки до реки

    Шаг 3. Назовите то, что мы ищем. Пусть время.
    Шаг 4. Переведите в уравнение, записав

    соответствующая формула. Заменить в данном

    информация.

    Шаг 5. Решите уравнение.
    Шаг 6.Отметьте ответ в задаче и сделайте

    уверен, что это имеет смысл.

    5 секунд кажутся разумной продолжительностью

    раз?

    Да.
    Шаг 7. Ответьте на вопрос. Для

    солнцезащитные очки, чтобы добраться до реки.

    Вертолет сбросил спасательный пакет с высоты 1296 футов. Используйте формулу, чтобы определить, сколько секунд потребовалось, чтобы пакет достиг земли.

    Мойщик окон сбросил ракель с платформы на высоте 196 футов над тротуаром. Используйте формулу, чтобы определить, сколько секунд прошло, чтобы ракель достиг тротуара.

    секунд

    Сотрудники полиции, расследующие автомобильные аварии, измеряют длину следов заноса на тротуаре. Затем они используют квадратные корни, чтобы определить скорость в милях в час, с которой машина ехала до того, как затормозила.

    Следы заноса и скорость автомобиля

    Если длина пятен заноса составляет d футов, то скорость с автомобиля до того, как были применены тормоза, может быть определена по формуле

    После автомобильной аварии следы заноса одной машины достигли 190 футов. Воспользуйтесь формулой, чтобы определить скорость автомобиля до того, как были задействованы тормоза. Округлите ответ до ближайшей десятой.

    Step 7 is to answer the question. The car was traveling approximately 67.5 mph before the brakes were applied.» data-label=»»>

    Шаг 1. Прочтите проблему
    Шаг 2. Определите , что мы ищем. скорость автомобиля
    Шаг 3. Назовите то, что мы ищем, Пусть скорость.
    Шаг 4. Переведите в уравнение, написав

    соответствующая формула.Замена в

    предоставлена ​​информация.

    Шаг 5. Решите уравнение.
    Округлить до 1 десятичного знака.
    Скорость автомобиля до торможения.

    был 67,5 миль в час.

    Следователь ДТП измерил следы заноса автомобиля.Длина следов заноса составляла 76 футов. Воспользуйтесь формулой, чтобы определить скорость автомобиля до того, как были задействованы тормоза. Округлите ответ до ближайшей десятой.

    футов

    Следы заноса автомобиля, попавшего в аварию, были длиной 122 фута. Используйте формулу, чтобы найти скорость автомобиля до того, как были задействованы тормоза. Округлите ответ до ближайшей десятой.

    футов

    Ключевые понятия

    • Биномиальные квадраты

    • Решите радикальное уравнение
      1. Выделите один из радикальных членов на одной стороне уравнения.
      2. Возвести обе части уравнения в степень индекса.
      3. Есть еще радикалы?

        Если да, повторите шаги 1 и 2 еще раз.

        Если нет, решите новое уравнение.

      4. Проверьте ответ в исходном уравнении.
    • Стратегия решения проблем для приложений с формулами
      1. Прочтите задачу и убедитесь, что все слова и идеи понятны. При необходимости нарисуйте фигуру и пометьте ее данной информацией.
      2. Определите, что мы ищем.
      3. Назовите то, что мы ищем, выбрав переменную для его представления.
      4. Переведите в уравнение, написав соответствующую формулу или модель для ситуации. Подставьте в данную информацию.
      5. Решите уравнение, используя хорошие методы алгебры.
      6. Проверьте ответ в задаче и убедитесь, что он имеет смысл.
      7. Ответьте на вопрос полным предложением.
    • Падающие предметы
      • На Земле, если объект падает с высоты h футов, время в секундах, которое потребуется, чтобы достичь земли, определяется по формуле
    • Следы заноса и скорость автомобиля
      • Если длина пятен скольжения составляет d футов, то скорость с автомобиля до того, как были применены тормоза, может быть определена по формуле
    Практика ведет к совершенству

    Решите радикальные уравнения

    В следующих упражнениях решите.

    Решите радикальные уравнения с двумя радикалами

    В следующих упражнениях решите.

    Использование радикалов в приложениях

    В следующих упражнениях решите. Округлите округление до одного десятичного знака.

    Ландшафтный дизайн Рид хочет иметь квадратный сад на заднем дворе. У него достаточно компоста, чтобы покрыть площадь в 75 квадратных футов. Используйте формулу, чтобы найти длину каждой стороны его сада. Округлите ответ до ближайшей десятой доли фута.

    футов

    Ландшафтный дизайн Винс хочет сделать квадратный внутренний дворик в своем дворе. У него достаточно бетона, чтобы вымостить площадь в 130 квадратных футов. Используйте формулу, чтобы найти длину каждой стороны его внутреннего дворика.Округлите ответ до ближайшей десятой доли фута.

    Gravity Дельтаплан сбросил свой мобильный телефон с высоты 350 футов. Используйте формулу, чтобы определить, сколько секунд потребовалось мобильному телефону, чтобы достичь земли.

    секунд

    Gravity Строительный рабочий уронил молот при строительстве пешеходной дорожки к Гранд-Каньону на высоте 4000 футов над рекой Колорадо. Воспользуйтесь формулой, чтобы узнать, сколько секунд потребовалось молоту, чтобы достичь реки.

    Расследование ДТП Следы заноса автомобиля, попавшего в аварию, достигли 216 футов. Воспользуйтесь формулой, чтобы определить скорость автомобиля до того, как были задействованы тормоза. Округлите ответ до ближайшей десятой.

    Расследование происшествий Следователь измерил следы заноса одного из автомобилей, попавших в аварию. Длина следов заноса составляла 175 футов. Используйте формулу, чтобы найти скорость автомобиля до того, как были задействованы тормоза.Округлите ответ до ближайшей десятой.

    Письменные упражнения

    Объясните, почему уравнение вида не имеет решения.

    ⓐ Решите уравнение

    ⓑ Объясните, почему одно из найденных «решений» на самом деле не было решением уравнения.

    Самопроверка

    ⓐ После выполнения упражнений используйте этот контрольный список, чтобы оценить свое мастерство в достижении целей этого раздела.

    ⓑ Что вы сделаете, изучив этот контрольный список, чтобы стать уверенным в достижении всех целей?

    Глоссарий

    радикальное уравнение
    Уравнение, в котором переменная находится в подкоренном выражении радикального выражения, называется радикальным уравнением.

    Как решать радикальные уравнения. Видеоурок и пошаговое занятие

    Как решить радикальные уравнения

    • 1) Изолировать радикал на одной стороне уравнения
    • 2) Возвести в квадрат обе части уравнения, чтобы исключить радикал
    • 3) Упростите и решите, как любые уравнения
    • 4) Подставьте ответы обратно в исходное уравнение, чтобы убедиться, что ваши решения верны (могут быть какие-то посторонние корни, которые не удовлетворяют исходному уравнению и которые вы должны выбросить).

    Видео ниже и наши примеры объясняют эти шаги, а затем вы можете попробовать наши практические задачи ниже.

    Видео о том, как решать радикальные уравнения

    Задача 1
    Задача 2
    Задача 3

    Шаг 1

    Шаг 1

    Шаг 2

    Шаг 2

    Шаг 4

    Подставьте ответ в исходное радикальное уравнение, чтобы убедиться, что ответ — действительное число.

    Шаг 4

    $$
    \ sqrt {3x -11} = 3x -x
    \\
    \ sqrt {3 (\ color {Red} {4}) -11} = 3 \ cdot (\ color {Red} {4}) — \ color {Red} {4}
    \\
    \ sqrt {1} = 8
    \\
    1 \ color {красный} {\ ne} 8
    $$

    Следовательно, отклонить 4 как решение, проверить 5 .

    $$
    \ sqrt {3x -11} = 3x -x
    \\
    \ sqrt {3 (\ color {Red} {5}) -11} = 3 (\ color {Red} {5}) — \ color {Red} {5}
    \\
    \ sqrt {15–11} = 15–5
    \\
    \ sqrt {15–11} = 15–5
    \\
    \ sqrt {4} = 10
    \\
    2 = 10
    \\
    \ color {красный} {\ ne} 10
    $$

    Следовательно, отклонить 5 как решение.

    Поскольку оба наших решения были отвергнуты, реальных решений этого уравнения нет.

    Радикальные уравнения | Суперпроф

    Из этой статьи вы узнаете, что такое радикальные уравнения и как их решать.Вы уже знакомы с уравнениями. Между уравнениями стоит знак равенства, а левая и правая части уравнений равны. Вы знаете, что такое радикальные уравнения?

    Что ж, прежде чем обсуждать радикальное уравнение, вы должны знать, что это за радикалы. Точно так же, как сложение и вычитание противоположны друг другу, а умножение является обратной операцией деления, радикал противоположен показателю степени. Радикальное выражение — это выражение, имеющее константу, коэффициент или переменную под символом корня

    .Например, и являются радикалами, поскольку содержат радикальный знак. Теперь давайте используем определение радикальных уравнений:

    Уравнение, в котором переменная находится под корнем или знаком радикала, известно как радикальное уравнение

    Радикальные уравнения также известны как иррациональные уравнения . Другими словами, можно сказать, что неизвестные значения в радикальных уравнениях присутствуют под знаком радикала. В следующем разделе мы узнаем, какие шаги необходимо предпринять для решения радикального уравнения.

    Вы можете найти репетитора по математике здесь.

    Шаги для решения радикального уравнения

    Для решения радикального уравнения необходимо выполнить следующие шаги:

    • Во-первых, вы должны изолировать радикальное выражение от уравнения, которое включает переменную. Если есть два или более радикальных выражения, включающих переменную, выделите одно из них.
    • В соответствии с индексом радикального выражения возвести обе части уравнения в индекс или в степень.Например, если радикальное выражение включает функцию квадратного корня, возьмите квадрат обеих частей уравнения.
    • Повторите шаги 1 и 2, если осталось радикальное выражение. Если радикального выражения не осталось, решите уравнение для неизвестного значения.
    • Подставьте неизвестное значение в исходное уравнение, чтобы проверить, верно ли уравнение для этого значения или нет.

    Помните, что иногда, возводя обе части уравнения в индекс, мы получаем значение, которое делает уравнение неверным.В математике эти решения известны как сторонние решения .

    В следующем разделе мы решим пару примеров, которые прояснят всю концепцию решения радикального уравнения.

    Лучшие репетиторы по математике

    Первый урок бесплатно

    Пример 1

    Решите следующее уравнение:

    Решение

    Чтобы решить уравнение, выполните следующие действия:

    Шаг 1. Изолируйте корень нам нужно изолировать радикал, взяв любую переменную или константу на другую сторону уравнения.В этом примере мы возьмем x в правую часть уравнения:

    Шаг 2 — Возьмите квадрат обеих сторон уравнения

    Поскольку в этом примере у нас есть функция квадратного корня, следовательно, на этом этапе мы возьмем квадрат обеих сторон уравнения:

    Шаг 3 — Решите уравнение для неизвестного значения

    Теперь у нас есть уравнение, мы можем легко решить его, чтобы найти значение of x:

    Вы можете использовать квадратную формулу для получения значения x или разложить на множители приведенное выше уравнение следующим образом:

    Шаг 4. Подставьте значение в исходное уравнение

    Чтобы проверить, верно ли ваше решение, мы должны подставить значение x в исходное уравнение, чтобы увидеть, верно ли уравнение для этого значения:

    900 18

    Пример 2

    Решите следующее уравнение:

    Решение

    Для решения уравнения выполните следующие шаги:

    Шаг 1. Выделите радикал

    Во-первых, нам нужно изолировать радикал, переводя любую переменную или константу на другую сторону уравнения.В этом примере мы возьмем -10 в правую часть уравнения:

    Шаг 2 — Возьмите квадрат обеих сторон уравнения

    Так как в этом примере у нас есть функция квадратного корня , следовательно, на этом шаге мы возьмем квадрат обеих сторон уравнения:

    Шаг 3 — Решите уравнение для неизвестного значения

    Теперь у нас есть уравнение, мы можем легко его решить, чтобы найти значение x:

    Шаг 4 — Подставьте значение в исходное уравнение

    Чтобы проверить, является ли ваше решение верным или нет, мы должны подставить значение x в исходное уравнение, чтобы увидеть, уравнение верно для этого значения или нет:

    Пример 3

    Решите следующее уравнение:

    Решение ция

    Выполните следующие шаги, чтобы решить уравнение:

    Шаг 1. Выделите радикал

    Во-первых, нам нужно изолировать радикал, взяв любую переменную или константу в другую часть уравнения.В этом примере мы возьмем -3 в правую часть уравнения:

    Шаг 2 — Возьмите квадрат обеих сторон уравнения

    Поскольку в этом примере у нас есть функция квадратного корня , поэтому на этом шаге мы возьмем квадрат обеих сторон уравнения:

    Шаг 3 — Решите уравнение для неизвестного значения

    Теперь у нас есть уравнение, мы можем легко его решить, чтобы найти значение x:

    Вы можете использовать квадратную формулу для получения значения x или разложить на множители приведенное выше уравнение следующим образом:

    Шаг 4 — Подставьте значение в исходное уравнение

    Чтобы проверить, верно ваше решение или нет, мы должны подставить значение x в исходное уравнение, чтобы увидеть, верно ли уравнение для этого значения:

    9 0018

    Мы знаем, что квадратный корень из 4 равен

    .В этом случае мы можем взять 2, чтобы уравнение стало истинным:

    Пример 4

    Решите следующее уравнение:

    Решение

    Выполните следующие шаги, чтобы решить уравнение:

    Шаг 1 — Выделите радикал

    Во-первых, нам нужно выделить радикал, взяв любую переменную или константу в другую часть уравнения. В этом примере мы возьмем — x + 3 в правую часть уравнения:

    Шаг 2 — Возьмите куб из обеих частей уравнения

    Поскольку у нас есть функция кубического корня в этом примере, следовательно, на этом этапе мы возьмем куб с обеих сторон уравнения:

    Шаг 3 — Решите уравнение для неизвестного значения

    Теперь у нас есть уравнение, которое мы можем легко решить чтобы найти значение x:

    Шаг 4. Подставьте значение в исходное уравнение

    Чтобы проверить, верно ли ваше решение, мы должны подставить значение x в исходное Уравнение, чтобы увидеть, верно ли уравнение для этого значения или нет:

    Пример 5

    Решите следующее уравнение:

    Решение

    9 0017 Выполните следующие шаги, чтобы решить уравнение:

    Шаг 1. Выделите радикал

    Во-первых, нам нужно изолировать радикал, взяв любую переменную или константу в другую часть уравнения.В этом примере мы возьмем + 2 в правую часть уравнения:

    Шаг 2 — Возьмите куб из обеих частей уравнения

    Поскольку в этом примере у нас есть функция кубического корня , поэтому на этом шаге мы возьмем куб из обеих частей уравнения:

    Шаг 3 — Решите уравнение для неизвестного значения

    Теперь у нас есть уравнение, мы можем легко его решить, чтобы найти значение x:

    Разделите обе стороны на 3:

    Шаг 4 — Подставьте значение в исходное уравнение

    Чтобы проверить, верно ли ваше решение, мы должны подставить значение x в исходном уравнении, чтобы проверить, верно ли уравнение для этого значения:

    Вычисление | Бесплатный полнотекстовый | Символьные вычисления для решения иррационального уравнения на основе метода симметричных многочленов

    2.1. Суть проблемы посторонних решений

    Продемонстрируем суть проблемы ложных решений иррациональных уравнений при использовании компьютерных математических пакетов.

    Рассмотрим иррациональное уравнение с общими параметрами:
    где z — переменная, a, b, c, d, f — действительные параметры, причем a и c одновременно не равны нулю и f≥0. Решив символьно уравнение (1) с использованием стандартных инструментов математической пакет Mathcad, мы получаем два корня (см. листинг программы Mathcad на рисунке 1).

    Обозначим найденные функции корней нескольких параметров как zr1 (a, d, c, d, f) и zr2 (a, d, c, d, f).

    Обратите внимание, что формулы, полученные для корней zr1 (a, d, c, d, f) и zr2 (a, d, c, d, f), не приводят к вариантам корней для случая a = c, поскольку для a = c Уравнение (1) имеет единственный корень из-за сокращения членов с одинаковыми коэффициентами (см. листинг программной части в Mathcad на рисунке 2).

    Аналогично, уравнение (1) имеет только один корень в случае a = 0 или c = 0.

    Мы будем рассматривать уравнение (1) в более общем случае a ≠ c.

    Анализ найденных корней zr1 (a, d, c, d, f) и zr2 (a, d, c, d, f) показывает, что они не удовлетворяют уравнению (1) для всех значений параметров. Чтобы продемонстрировать это, мы вводим величины ∆1 (a, d, c, d, f) и ∆2 (a, d, c, d, f), которые представляют собой разности между левой и правой частями уравнения (1). где в переменную z подставляются значения найденных корней zr1 (a, d, c, d, f) и zr2 (a, d, c, d, f) соответственно. Очевидно, что если корни zr1 (a, d, c, d, f) и zr2 (a, d, c, d, f) удовлетворяют уравнению (1), то значения ∆1 (a, d, c, d , f) = 0 и ∆2 (a, d, c, d, f) = 0.

    Построим график ∆1 (a, d, c, d, f) и ∆2 (a, d, c, d, f) как функции параметра f для a = 2, b = 1, c = ± 4, d = 1000 (см. рисунок 3a, b). Рисунок 3a, b показывает, что корни zr1 (a, d, c, d, f) и zr2 (a, d, c, d, f) не удовлетворяют уравнению (1 ) для всех значений параметра f. Следовательно, при c = −4 (рисунок 3a) корень zr1 (a, d, c, d, f) удовлетворяет (1) только при f≥22.4, а корень zr2 (a, d, c, d, f ) удовлетворяет при f≥31,6. При c = 4 (рисунок 3b) корень zr2 (a, d, c, d, f) удовлетворяет (1) только при f≥31.6, в то время как корень zr1 (a, d, c, d, f) не удовлетворяет вообще удовлетворяют (1).

    Таким образом, с помощью математических пакетов можно констатировать появление посторонних корней в символьном решении иррационального уравнения вида (1).

    2.3. Применение предложенного метода решения электрической задачи

    Поясним суть проблемы сторонних решений на примере прикладной электрической задачи. Требуется найти оптимальное распределение полной мощности P = P1 + P2 между двумя пользователями емкостей P1 и P2 с требуемым общим напряжением U = U1 + U2, меньшим, чем напряжение пробоя.Сопротивления пользователей равны соответственно R1 и R2.

    Согласно теории электрических цепей, выражения для мощностей описываются формулами [12]:

    P1 = U12R1, P2 = U22R2.

    (7)

    Затем, задав в качестве параметров сопротивления R1 и R2, полное напряжение U и полную мощность P, выводится иррациональное уравнение для искомой переменной P1:

    P1 · R1 + (P − P1) · R2 = U.

    (8)

    Решая символьно уравнение (8) с помощью стандартных инструментов математического пакета Mathcad, мы получаем два корня (см. Листинг программы Mathcad на рисунке 5).Можно отметить, что при выполнении вычислений с использованием пакетов Mathematica или Matlab эквивалентные корни получаются (см. Списки программ в пакетах Wolfram Mathematica [13] и Matlab [14,15] на рисунке 6).

    Найденные корни являются функциями нескольких параметров Pr1 (U, P, R1, R2) и Pr2 (U, P, R1, R2).

    После присвоения параметрам следующих значений P = 8000, R1 = 1, R2 = 2, корни уравнения (8) Pr1 (U, 8000, 1,2) и Pr2 (U, 8000, 1,2) равны график как функция требуемого полного напряжения U (рисунок 7).Давайте проверим, действительно ли функции Pr1 (U, 8000, 1, 2) и Pr2 (U, 8000, 1, 2) являются решениями уравнения (8) для любого значения U. Для этого мы строим график правой части уравнения (8) — просто Y (U) = U, и график левой части уравнения (8) путем подстановки в него функций Pr1 (U, 8000, 1, 2) и Pr2 (U, 8000, 1, 2) вместо искомой переменной P1 (рисунок 8). Очевидно, что решения Pr1 (U, 8000, 1, 2) и Pr2 (U, 8000, 1, 2) являются истинными решениями задачи только для тех значений U, где графики левой и правой частей совпадают.

    По графику:

    • При U <89,4 оба решения P r1 (U, 8000, 1, 2) и P r2 (U, 8000, 1, 2) ложны;

    • Для 89,4 ≤ U <126,5 решение P r1 (U, 8000, 1, 2) ложно, а P r2 (U, 8000, 1, 2) верно;

    • При U ≥ 126,5 верны оба решения P r1 (U, 8000, 1, 2) и P r2 (U, 8000, 1, 2).

    Таким образом, мы убедились, что решение уравнения (8), показанное на рисунке 5, включает посторонние корни.

    2.3.1. Реализация предложенной методики

    Воспользуемся предложенной методикой для устранения посторонних корней. Требуется определить неравенства, определяющие диапазон допустимых значений параметра — полное напряжение U.

    Для определения диапазона допустимых значений параметра U было предложено использовать свойства симметричных многочленов. Обратите внимание, что рассматриваемое здесь уравнение (8) не является симметричным относительно переменных P1, P2. Таким образом, графики корней Pr1 (U, 8000, 1, 2) и Pr2 (U, 8000, 1, 2) также не симметричны (рисунок 7).Введем обозначения:

    x = P1 · R1, y = (P − P1) · R2, σ1 = x + y, σ2 = x · y.

    (9)

    Уравнение (8) принимает следующий вид:

    σ12 = P · R2 + (R1 − R2) · P1 + 2 · σ2, σ1 = U,

    (10)

    из этого происходит:

    σ2 = U2 −P · R2− (R1 − R2) · P12.

    (11)

    Поскольку радикалы должны быть неотрицательными, σ1≥0 и σ2≥0.

    Поскольку σ1 = U, требование σ1≥0 проверяется для U≥0.

    Из условия σ2≥0 следует диапазон допустимых значений параметра U:

    U≥P · R2 + (R1 − R2) · P1.

    (12)

    Сравнивая графики корней на рисунке 8 и график функции σ2 (U) на рисунке 9 для конкретных значений P, R1, R2, P1, мы замечаем, что нижняя граница для значения U = 89,4 соответствует минимально допустимое значение переменной P1 = 0, а точка появления второго корня U = 126,5 соответствует максимально допустимому значению P1 = P = 8000. Наконец, решаем систему, заданную уравнением (8) и неравенством (12). ):

    P1 · R1 + (P − P1) · R2 = U, U≥P · R2 + (R1 − R2) · P1.

    (13)

    График правильного аналитического решения, полученного с помощью Mathcad, показан на рисунке 10. Сравнивая его с графиком на рисунке 7, мы видим, что посторонние корни исключаются и, как отмечалось ранее, в интервале 89,4≤U <126,5 Уравнение ( 8) имеет только один корень Pr (U, 8000, 1, 2), а в интервале U≥126,5 - два корня Pr1 (U, 8000, 1, 2) и Pr2 (U, 8000, 1, 2).

    2.3.2. Частный случай

    Давайте рассмотрим частный случай уравнения (8), когда пользовательские сопротивления равны R1 = R2.В этом случае левая часть уравнения (8) уже изначально является симметричным многочленом относительно переменных P1, P2:

    P1 · R1 + (P − P1) · R1 = U.

    (14)

    Графики решения (14), представленные на рисунке 11 и аналогичные графикам на рисунке 7, имеют симметричный вид. Система (13) в этом случае сводится к виду:

    P1 · R1 + (P − P1) · R1 = U, U≥P · R1.

    (15)

    Общий вид аналитического решения системы (15), полученного с помощью Mathcad, показан на рисунке 12.Аналитическое решение записывается с использованием оператора условного if, который позволяет определить другой тип решения для разных диапазонов значений U. График правильного аналитического решения системы (15), показанный на рисунке 13, очевидно, имеет симметричный вид, т.е. на всем допустимом интервале U≥126,5 имеется два симметричных корня Pr1 (U, 8000, 1, 2) и Pr2 (U, 8000, 1, 2).

    Таким образом, в этом разделе мы описали метод получения правильного аналитического решения иррационального уравнения с использованием свойств симметрии, продемонстрировав его на примере решения уравнения (8).

    Квадраты и квадратные корни — решение радикальных уравнений

    Теорема Пифагора отлично подходит для решения задач со словами. Это также хорошо для зажигательной беседы за ужином. По крайней мере, мы, так думаем; от того чувака, который с криком выбежал из комнаты, еще нет ни слова. Когда слово «проблема» описывает ситуацию, которую можно визуализировать с помощью треугольника, вероятно, где-то поблизости таится теорема Пифагора.

    Даже Дарт Вейдер, классический скрытень, знает это.

    Пример задачи

    Хуан и Ленор встретились за обедом. В 13:00 они расстались. Может быть, навсегда, учитывая, как они все оставили. Хуан ехал на юг со скоростью 30 миль в час, а Ленор ехала на восток со скоростью 60 миль в час. Видимо, она была расстроена больше, чем он. В 13:30, как далеко друг от друга Хуан и Ленор?

    Здесь нужна картинка, и определенно где-то здесь есть треугольник. Хуан ехал на юг или прямо вниз, а Ленор ехала на восток или прямо вправо:

    Вопрос просит нас найти c , и нам дали достаточно информации, чтобы найти a и б .Эти двое путешествуют уже полчаса … надеюсь, достаточно долго, чтобы остыть и пожалеть о том, что бросили всю эту мебель.

    Ленор путешествовала.

    Хуан путешествовал.

    Через полчаса треугольник будет выглядеть так:

    Мы должны найти расстояние между Ленор и Хуаном, поэтому нам нужно найти c . Мы можем сделать это с помощью теоремы Пифагора:

    (15) 2 + (30) 2 = c 2
    225 + 900 = c 2
    1125 = c 2

    Теперь извлеките квадратный корень:

    Отрицательное расстояние не имеет смысла, хотя они находились практически на отрицательном расстоянии друг от друга полчаса назад, когда были в глотке.Таким образом, мы берем только положительный квадратный корень и заключаем, что в 13:30 Хуан и Ленор находятся на расстоянии нескольких миль (около 33,5 миль).

    Для указанной выше проблемы, вероятно, приемлемым ответом будет 33,5 мили. Какая разница, если Хуан и Ленор находятся ближе к 33,541 милям друг от друга? Однако мы дали ответ, потому что это точный ответ . Мы можем не знать всех десятичных знаков числа, но мы точно знаем, как далеко они находятся друг от друга в 13:30.

    Если проблема требует дать точный ответ, а в вашем ответе есть радикалы, оставьте радикалов там.Точный ответ на вопрос «каков квадратный корень из 2?» есть, а не 1.414. Возможно, вы не удивитесь, узнав, что математика — это что-то вроде точной науки, поэтому эти точные ответы важны.

    Примечание. С момента написания этого примера мы в Shmoop рады сообщить, что Хуан и Ленор снова собрались вместе, поговорили и уладили проблемы. При этом не было брошено ни одного складного стула.

    Пример задачи

    Найдите точную площадь прямоугольного треугольника с гипотенузой длины 20 и одним катетом длины 7.

    Лучше нарисовать картинку, отчасти потому, что мы ищем повод, чтобы растянуть свои художественные мускулы. Используйте b для неизвестного отрезка и сделайте основание треугольника:

    Мы знаем, что площадь прямоугольного треугольника равна.

    Другими словами, площадь прямоугольного треугольника равна половине произведения длин его сторон. Мы знаем длину одной ноги, и если бы у нас была длина другой ноги, все было бы готово. К счастью, мы можем использовать теорему Пифагора, чтобы найти длину этого недостающего фрагмента.В противном случае этот треугольник, возможно, потребуется приспособить для опоры колышка.

    Мы знаем, что b 2 + 7 2 = 20 2 , поэтому b 2 = 351.

    Получение положительного квадратного корня (поскольку треугольник не может иметь отрицательного основания ), видим, что:

    Это здорово. Однако если вы запишете это в качестве окончательного ответа, вся ваша работа будет напрасной. Мы еще не ответили на актуальный вопрос проблемы.Оглядываясь назад на проблему, чтобы напомнить себе, что мы должны были искать (о, верно, область треугольника), мы видим, что теперь у нас есть все, что нам нужно. Мы знаем, что высота треугольника равна 7, а его основание равно:

    Это немного неприятно выглядит, но, к сожалению, нет лучшего способа написать это. Были сделаны. Мы умываем руки от вас, некрасивая проблема слова.

    Будьте осторожны: Если вы прочитали примеры в этом разделе, и они имели для вас полный смысл, похлопайте себя по спине или по колену, если это более доступно.У вас отличный старт.

    Затем спросите себя: знаете ли вы, что делать, если бы вам предложили аналогичные проблемы и попросили решить их самостоятельно, не просматривая веб-страницы, книги или заметки? Да, вот в чем загвоздка. Прежде чем решить, что вы готовы прекратить учебу, убедитесь, что вы знаете, как подходить к упражнениям и другим задачам, указанным в учебнике или у учителя. Если следующие упражнения вызывают у вас проблемы, вы всегда можете просмотреть приведенные выше примеры, чтобы выяснить, где вы ошиблись.Они всегда будут рядом с вами, как Росс, Рэйчел, Чендлер, Моника, Джоуи и Фиби. Хорошо, мы бросим туда и Гюнтера.

    Вот подсказка для всех задач, возникающих в разделе упражнений: нарисуйте картинки. Бонусный совет: картинки должны действительно относиться к решаемой проблеме. Арт-класс — это следующий период.

    6.3 Решение радикальных уравнений — промежуточная алгебра

    Цели обучения

    • Решите радикальные уравнения
      • Выделить радикальный член в уравнении с помощью алгебры и правил для радикалов и показателей
      • Определите радикальное уравнение без решений или посторонних решений
    • Приложения с радикальными уравнениями
      • Кинетическая энергия
      • Объем
      • Свободное падение

    Теперь, когда вы понимаете, как комбинировать радикальные члены и выражения с помощью алгебраических операций, вы можете решать уравнения, содержащие радикалы.Вскоре мы сможем помочь Джоан (и Анне) выяснить, какой высоты должна быть лестница! В этом уроке вы будете использовать свойства равенства сложения и умножения, а также правила для показателей.

    Уравнение, которое содержит радикальное выражение , называется радикальным уравнением . Решение радикальных уравнений требует применения правил экспонент и некоторых основных алгебраических принципов. В некоторых случаях это также требует поиска ошибок, вызванных возведением неизвестных величин в четную степень.

    Выделите радикальный термин

    Основная стратегия решения радикальных уравнений состоит в том, чтобы сначала выделить радикальный член, а затем возвести обе части уравнения в степень, чтобы удалить радикал. (Причина использования степеней станет понятной через мгновение.) Это тот же тип стратегии, который вы использовали для решения других, нерадикальных уравнений: измените выражение, чтобы изолировать переменную, которую вы хотите знать, а затем решите полученное уравнение .

    Есть две ключевые идеи, которые вы будете использовать для решения радикальных уравнений.{2}} = х \). (Это свойство позволяет вам «удалить» радикалы из ваших уравнений.)

    Начнем с радикального уравнения, которое можно решить за несколько шагов: \ (\ sqrt {x} -3 = 5 \). {2}} = x \).{2}} \\ x = 64 \ end {array} \)

    Ответ

    \ (x = 64 \) — это решение \ (\ sqrt {x} -3 = 5 \)

    Чтобы проверить свое решение, вы можете заменить x на 64 в исходном уравнении. Имеет ли \ (\ sqrt {64} -3 = 5 \)? Да, квадратный корень из 64 равен 8 и \ (8-3 = 5 \).

    Обратите внимание, как вы объединили одинаковые члены, а затем возводили в квадрат обе стороны уравнения в этой задаче. Это стандартный метод удаления радикала из уравнения. Важно выделить радикал на одной стороне уравнения и максимально упростить перед возведением в квадрат .Чем меньше членов будет перед возведением в квадрат, тем меньше дополнительных членов будет сгенерировано в процессе решения.

    В приведенном выше примере только переменная x находилась под радикалом. Иногда вам нужно будет решить уравнение, которое содержит несколько членов под радикалом. Выполните те же шаги, чтобы решить их, но обратите внимание на критический момент: возведите в квадрат обе стороны уравнения, а не отдельные члена . Посмотрите, как решаются следующие две проблемы.{2}} = х + 8 \). Теперь упростите уравнение и решите относительно x .

    \ (\ begin {array} {r} x + 8 = 9 \\ x = 1 \ end {array} \)

    Проверьте свой ответ. Подстановка 1 вместо x в исходном уравнении дает истинное утверждение, поэтому решение является правильным.

    \ (\ begin {array} {r} \ sqrt {1 + 8} = 3 \\\ sqrt {9} = 3 \\ 3 = 3 \ end {array} \)

    Ответ

    \ (x = 1 \) является решением \ (\ sqrt {x + 8} = 3 \).

    В следующем видео-примере вы увидите еще два примера, похожих на приведенные выше.{2}} \, \, \, \ end {array} \)

    Упростите уравнение и решите относительно x .

    \ (\ begin {array} {r} 2x + 3 = 25 \\ 2x = 22 \\ x = 11 \ end {array} \)

    Проверьте свой ответ. Подстановка 11 вместо x в исходном уравнении дает истинное утверждение, поэтому решение является правильным.

    \ (\ begin {array} {r} 1+ \ sqrt {2 (11) +3} = 6 \\ 1+ \ sqrt {22 + 3} = 6 \\ 1+ \ sqrt {25} = 6 \ \ 1 + 5 = 6 \\ 6 = 6 \ end {array} \)

    Ответ

    \ (x = 11 \) — решение для \ (1+ \ sqrt {2x + 3} = 6 \).

    Решение радикальных уравнений

    Чтобы решить радикальные уравнения, выполните следующие четыре шага.{2} \).

  1. Как только радикал удален, ищите неизвестное.
  2. Отметьте все ответы.

Определите радикальное уравнение без решений или посторонних решений

Следование правилам важно, но не менее важно уделять внимание математике, стоящей перед вами, особенно при решении радикальных уравнений. Взгляните на следующую задачу, которая демонстрирует потенциальную ловушку квадратуры обеих сторон для удаления радикала.

Пример

Решить.{2}} \)

Напишите упрощенное уравнение и решите относительно a .

\ (\ begin {array} {r} a-5 = 4 \\ a = 9 \ end {array} \)

Теперь проверьте решение, подставив \ (a = 9 \) в исходное уравнение.

Это не проверяет!

\ (\ begin {array} {r} \ sqrt {9-5} = — 2 \\\ sqrt {4} = — 2 \\ 2 \ ne -2 \ end {array} \)

Ответ

Нет решения.

Посмотрите на это: ответ \ (a = 9 \) не дает истинного утверждения, если подставить его обратно в исходное уравнение.Что случилось?

Проверьте исходную проблему: \ (\ sqrt {a-5} = — 2 \). Обратите внимание, что радикал установлен равным \ (- 2 \), и вспомните, что главный квадратный корень из числа может быть только положительным . Это означает, что никакое значение для a не приведет к радикальному выражению, положительный квадратный корень которого равен \ (- 2 \)! Вы могли заметить это сразу и сделать вывод, что для и решений не существует. Но почему процесс возведения в квадрат дал ответ \ (a = 9 \), который оказался неверным?

Ответ заключается в том, что возводит в квадрат само .{4} \) оба равны 16. Итак, когда вы возводили в квадрат -2 и получили 4 в этой задаче, вы искусственно превратили количество в положительное. Вот почему вам все же удалось найти значение для a : вы решили проблему, как если бы вы решали \ (\ sqrt {a-5} = 2 \)! (Правильное решение для \ (\ sqrt {a-5} = — 2 \) на самом деле «не решение».)

Неправильные значения переменной, такие как те, которые вводятся в результате процесса возведения в квадрат, называются посторонними решениями . Посторонние решения могут выглядеть как реальное решение, но вы можете идентифицировать их, потому что они не создадут истинного утверждения, когда будут подставлены обратно в исходное уравнение.Это одна из причин, почему проверка вашей работы так важна: если вы не проверяете свои ответы, подставляя их обратно в исходное уравнение, вы можете вводить в проблему посторонние решения.

В следующем видео мы представляем больше примеров решения радикальных уравнений путем выделения радикального члена с одной стороны.

Может быть трудно понять, почему вообще существуют посторонние решения. Обдумывание посторонних решений с помощью графического представления уравнения может помочь вам разобраться в происходящем.{2}} + 8x + 16 = x + 10 \). Квадрат с обеих сторон, возможно, привел к постороннему решению.

Вы можете построить график \ (x + 4 = \ sqrt {x + 10} \) на координатной плоскости, разбив его на систему двух уравнений: \ (y = x + 4 \) и \ (y = \ sqrt { х + 10} \). График показан ниже. Обратите внимание, как два графика пересекаются в одной точке, когда значение x равно \ (- 1 \). Это значение x , которое удовлетворяет обоим уравнениям, поэтому оно является решением системы.

Теперь, следуя работе, которую мы проделали в примере задачи, давайте возведем оба выражения в квадрат, чтобы удалить переменную из радикала.{2} + 8x + 16 \) и \ (y = x + 10 \) показаны ниже. Обратите внимание, как два графика пересекаются в двух точках, когда значения x равны \ (- 1 \) и \ (- 6 \).

Хотя \ (x = -1 \) показано как решение на обоих графиках, возведение обеих сторон уравнения в квадрат привело к добавлению постороннего решения, \ (x = -6 \). Опять же, вот почему так важно проверять свои ответы при решении радикальных уравнений!

Пример

Решить. \ (4+ \ sqrt {x + 2} = x \)

Показать решение
Выделите радикальный термин.{2}} — 9x + 14 \ end {array} \)

Этот многочлен также может быть выражен как произведение двух двучленов: \ ((x-7) \ cdot (x-2) \).

\ (\ begin {array} 0 = \ left (x-7 \ right) \ left (x-2 \ right) \ end {array} \)

Чтобы продукт был равен нулю, по крайней мере, один из двух факторов должен быть равен нулю. Установите каждый коэффициент равным нулю и решите для x.

\ (\ begin {array} {c} \ left (x-7 \ right) = 0 \ text {или} \ left (x-2 \ right) = 0 \\ x = 7 \ text {или} x = 2 \ end {array} \) \ (\)

Теперь проверьте оба решения, подставив их в исходное уравнение.

\ (\ begin {array} {r} 4+ \ sqrt {7 + 2} = 7 \\ 4+ \ sqrt {9} = 7 \\ 4 + 3 = 7 \\ 7 = 7 \\\ text { ИСТИНА!} \\\\ 4+ \ sqrt {2 + 2} = 2 \\ 4+ \ sqrt {4} = 2 \\ 4 + 2 = 2 \\ 6 = 2 \\\ text {FALSE!} \ конец {массив} \)

Поскольку \ (x = 2 \) дает ложное утверждение, это решение является посторонним.

Ответ

\ (x = 7 \) — единственное решение.

В следующем видео мы показываем еще один пример радикальных уравнений, которые имеют посторонние решения. Всегда не забывайте проверять свои ответы на наличие радикальных уравнений.

В нашем последнем примере мы будем использовать разность квадратов для решения радикального уравнения с двумя радикалами.Вы увидите, что мы должны сначала изолировать и устранить один радикал, а затем заняться изоляцией и устранением другого. Мы также будем использовать методы факторизации для трехчленов и принцип нулевого произведения, чтобы решить это уравнение.

Пример

Решите \ (\ sqrt {2x + 3} + \ sqrt {x — 2} = 4 \).

Покажи ответ

Поскольку это уравнение содержит два радикала, мы выделяем один радикал, удаляем его, а затем выделяем второй радикал.

\ (\ begin {array} {ll} \ sqrt {2x + 3} + \ sqrt {x — 2} \ hfill & = 4 \ hfill & \ hfill \\ \ sqrt {2x + 3} \ hfill & = 4- \ sqrt {x — 2} \ hfill & \ text {Subtract} \ sqrt {x — 2} \ text {с обеих сторон}.{2} -86x + 249 = 0 \ hfill & \ hfill \\ \ left (x — 3 \ right) \ left (x — 83 \ right) = 0 \ hfill & \ text {Разложить на множители и решить}. \ Hfill \ \ x = 3 \ hfill & \ hfill \\ x = 83 \ hfill & \ hfill \ end {array} \)

Предлагаемые решения: \ (x = 3 \) и \ (x = 83 \). Проверьте каждое решение в исходном уравнении.

\ (\ begin {array} {l} \ sqrt {2x + 3} + \ sqrt {x — 2} \ hfill & = 4 \ hfill \\ \ sqrt {2x + 3} \ hfill & = 4- \ sqrt {x — 2} \ hfill \\ \ sqrt {2 \ left (3 \ right) +3} \ hfill & = 4- \ sqrt {\ left (3 \ right) -2} \ hfill \\ \ sqrt {9} \ hfill & = 4- \ sqrt {1} \ hfill \\ 3 \ hfill & = 3 \ hfill \ end {array} \)

Одно из решений — \ (x = 3 \).

Чек \ (x = 83 \).

\ (\ begin {array} {l} \ sqrt {2x + 3} + \ sqrt {x — 2} \ hfill & = 4 \ hfill \\ \ sqrt {2x + 3} \ hfill & = 4- \ sqrt {x — 2} \ hfill \\ \ sqrt {2 \ left (83 \ right) +3} \ hfill & = 4- \ sqrt {\ left (83 — 2 \ right)} \ hfill \\ \ sqrt {169} \ hfill & = 4- \ sqrt {81} \ hfill \\ 13 \ hfill & \ ne -5 \ hfill \ end {array} \)

Единственное решение — \ (x = 3 \). Мы видим, что \ (x = 83 \) — постороннее решение.

Ответ

Единственное решение: \ (x = 3 \)

В нашем последнем видео мы показываем решение еще одного радикального уравнения, которое имеет квадратные корни с обеих сторон уравнения.Обратите особое внимание на то, как вы возводите в квадрат сторону, которая имеет сумму радикального члена и константы.

Численное вычисление радикалов

Обычно значение квадратного корня или другого радикала можно найти с помощью калькулятора или компьютера, но на удивление легко вычислить радикалы вручную. Простой итерационный метод вычисления квадратных корней был известен древним вавилонянам (1500 г. до н.э.) и грекам (100 г. н.э.). Этот вавилонский метод (иногда называемый методом Героя) был обобщен Исааком Ньютоном много веков спустя.2 = 25 \) и \ (16 \ le 23 \ le 25 \). Для простоты мы будем использовать 4.5 в качестве отправной точки.

Если \ (x = 4.5 \) было квадратным корнем, то \ (23 \ div 4.5 \) должно быть точно равно 4.5, но поскольку \ (\ sqrt {23} \) равно , а не точно 4.5, наше приближение имеет некоторую ошибка в нем: \ (23 \ div 4.5 = 5.11111 \).

Если бы у нас было точное значение квадратного корня, то 4,5 и 5,11111 точно совпадали. Согласно вавилонскому методу, следующее значение квадратного корня должно быть посередине между 4.5 и 5.11111 или 4.80556.

Теперь мы просто повторяем процесс. Если \ (x = 4.80556 \) был квадратным корнем, то \ (23 \ div 4.80556 \) должен в точности равняться 4.80556, но в нашем приближении есть некоторая ошибка: \ (23 \ div 4.80556 = 4.78612 \). Наше новое значение находится посередине между этими двумя, или 4,79584.

Повторите процесс еще раз, чтобы проверить наш ответ: \ (23 \ div 4.79584 = 4.79582 \). В нашем приближении все еще есть некоторая ошибка, но теперь эта ошибка меньше четвертого десятичного знака. Наше приближение квадратного корня из 23 равно 4.7958.

Модифицированная версия того же процесса может использоваться для аппроксимации кубических корней, корней n-й степени или любой другой гладкой функции. Все без калькулятора.

Приложения с радикальными уравнениями

Кинетическая энергия

Один из способов измерить количество энергии, которым обладает движущийся объект (например, автомобиль или американские горки), — это определить его кинетическую энергию. Кинетическая энергия (\ (E_ {k} \), измеряемая в Джоулях) объекта зависит от массы объекта ( м , измеряется в кг) и скорости ( v , измеряется в метрах в секунду).Скорость объекта можно записать как \ (v = \ sqrt {\ frac {2 {{E} _ {k}}} {m}} \).

Пример

Какова кинетическая энергия объекта массой 1000 кг, движущегося со скоростью 30 метров в секунду?

Показать решение
Определите переменные и известные значения.

\ (\ begin {array} {l} E_ {k} = \ text {unknown} \\\, \, m = 1000 \\\, \, \, \, v = 30 \ end {array} \)

Подставить значения в формулу.

\ (30 = \ sqrt {\ frac {2 {{E} _ {k}}} {1,000}} \)

Решите радикальное уравнение относительно E k .{2}} \\\\\, \, \, \, \, \, \, \, \, \, \, \, \, \, \, \, \, \, \, 900 = \ frac {2 {{E} _ {k}}} {1,000} \\\\ 900 \ cdot 1,000 = \ frac {2 {{E} _ {k}}} {1,000} \ cdot 1,000 \\\\\, \, \, \, \, \, \, \, 900 000 = 2 {{E} _ {k}} \\\\\, \, \, \, \, \, \, \, \ frac {900 000 } {2} = \ frac {2 {{E} _ {k}}} {2} \\\\\, \, \, \, \, \, \, \, \, \, 450 000 = {{ E} _ {k}} \ end {array} \)

Теперь проверьте решение, подставив его в исходное уравнение.

\ (\ begin {array} {l} 30 = \ sqrt {\ frac {2 \ cdot 450,000} {1,000}} \\ 30 = \ sqrt {\ frac {900,000} {1,000}} \\ 30 = \ sqrt {900} \\ 30 = 30 \ end {array} \)

Ответ

Кинетическая энергия составляет 450 000 джоулей.

Вот еще один пример определения кинетической энергии движущегося объекта.

Том

Муравьи-комбайны, обитающие на юго-западе США, создают обширную сеть туннелей для своих гнезд. В результате всех этих раскопок очень распространенной наземной отличительной чертой муравейников-комбайнов является насыпь конической формы из мелкого гравия или песка

Радиус конуса, высота которого в два раза больше его радиуса, определяется как: \ (r = \ sqrt [3] {\ frac {3V} {2 \ pi}} \).

Пример

Насыпь из гравия имеет форму конуса высотой, равной удвоенному радиусу. Вычислите объем такой насыпи из гравия радиусом 3,63 фута. Используйте \ (\ pi = 3,14 \).

Покажи ответ

Радиус конуса с учетом его объема можно найти по следующей формуле: \ (r = \ sqrt [3] {\ frac {3V} {2 \ pi}} \), \ (r \ ge 0 \\\ )

Куб с обеих сторон, чтобы удалить кубический корень. {2} \).2 \\\ pm \ sqrt {\ frac {h-600} {- 16}} = t \ end {array} \)

На этом давайте остановимся и поговорим о том, имеет ли смысл включать оба \ (\ pm \ sqrt {\ frac {h-600} {- 16}} \). Мы хотим, чтобы время было только положительным, поскольку мы говорим об измеримой величине, поэтому мы ограничим наши ответы только \ (+ \ sqrt {\ frac {h-600} {- 16}} \)

Мы хотим знать, в какое время высота будет 400 футов, поэтому мы можем заменить 400 на h.

\ (\ begin {array} {ccc} \ sqrt {\ frac {400-600} {- 16}} = t \\\ sqrt {\ frac {-200} {- 16}} = t \\\ sqrt { 12.5} = t \\ 3.54 = t \ end {array} \)

Ответ

Объект находится на высоте 400 футов за 3,54 секунды.

Анализ решения

Мы решили ограничить подкоренное выражение радикальных выражений неотрицательными числами. В предыдущем примере мы разделили на отрицательное число, а затем извлекли квадратный корень, чтобы найти t. В этом примере можно получить отрицательное число в подкоренном выражении? Другими словами, для каких значений высоты у нас возникнет проблема, когда мы можем извлечь квадратный корень из отрицательного числа? Чтобы ответить на этот вопрос, мы можем использовать алгебру.

Давайте переведем наш вопрос в неравенство. Опять же, при каких значениях h мы получим отрицательную величину под радикалом?

Подкоренное выражение \ (\ frac {h-600} {- 16} \), поэтому, если мы установим неравенство, мы можем решить его для h:

\ (\ begin {array} {ccc} \ frac {h-600} {- 16} \ lt0 \\ — 16 \ cdot \ frac {h-600} {- 16} \ lt0 \ cdot {-16} \\ h-600 \ gt0 \\ h \ gt600 \ end {array} \)

Мы можем интерпретировать это как «когда высота больше 600 футов, подкоренное выражение будет отрицательным и, следовательно, не будет действительным числом.«Если вы перечитаете вопрос, то увидите, что высота больше 600 не имеет смысла, потому что объект начинается на высоте 600 футов и падает на землю, поэтому высота уменьшается. Понимание того, в какой области находятся наши переменные, важно в задачах приложений, чтобы мы могли получить разумные ответы.

Радикальные уравнения играют важную роль в науке, технике и даже в музыке. Иногда вам может потребоваться использовать то, что вы знаете о радикальных уравнениях, для решения различных переменных в задачах такого типа.

Сводка

Распространенным методом решения радикальных уравнений является возведение обеих частей уравнения в любую степень, чтобы исключить радикальный знак из уравнения. Но будьте осторожны: когда обе части уравнения возводятся в степень и даже , существует вероятность того, что будут введены посторонние решения. Решая радикальное уравнение, важно всегда проверять свой ответ, подставляя значение обратно в исходное уравнение. Если вы получите истинное утверждение, то это значение является решением; если вы получите ложное утверждение, то это значение не является решением.


.

Добавить комментарий

Ваш адрес email не будет опубликован. Обязательные поля помечены *